You are on page 1of 70

CASE NO.

1232 - TIPOE
ART. XII SEC. 2: REGALIAN DOCTRINE; PUBLIC DOMAIN
LEE HONG HOK V. DAVID [G.R. NO. L-30389. DECEMBER 27, 1972.]

FACTS: Petitioners’ complaint is to have the Torrens Title of respondent David, as


Secretary of Agriculture, declared null and void for the reason that land not otherwise
passing into private ownership may not be disposed of by the state.

ISSUE: W/N such contention is a valid ground to declare the title null and void.

RULING: NO. The actual sale and award to David were not clandestine but open and
public official acts of an officer of the Government. Such contention is at war with settled
principles on public domain and the Regalian Doctrine under Article XII Sec. 2. of our
Constitution.

MAIN POINT: As far as the Philippines was concerned, "Spain in its earlier decrees
embodied the universal feudal theory that all lands were held from the Crown . . ." That
was a manifestation of the concept of jura regalia, which was adopted by the present
Constitution but stripped of regalian overtones: ownership is vested in the State, not in
the head of the State. Failure to show right or title to a land presumes that such remains
part of the public domain.

CASE NO. 1233- CHIONG


ARTICLE XII: SEC. 2 - REGALIAN DOCTRINE
CARIÑO VS INSULAR GOVERNMENT, 41 PHIL 935

FACTS:
An Igorot applied for the registration of a certain land. He and his ancestors had held the
land as owners for more than 50 years, which he inherited under Igorot customs. There
was no document of title issued for the land when he applied for registration. The
government contends that the land in question belonged to the state. Under the Spanish
Law, all lands belonged to the Spanish Crown except those with permit private titles.
Moreover, there is no prescription against the Crown.

ISSUE:
Whether or not the land in question belonged to the Spanish Crown under the Regalian
Doctrine.

RULING:
No. Law and justice require that the applicant should be granted title to his land. The
United States Supreme Court, through Justice Holmes declared: “It might perhaps, be
proper and sufficient to say that when, as far as testimony or memory goes, the land has
been held by individuals under a claim of private ownership, it will be presumed to have
been held in the same way from before the Spanish conquest, and never to have been
public land.” There is an existence of native title to land, or ownership of land by Filipinos
by virtue of possession under a claim of ownership since time immemorial and
independent of any grant from the Spanish Crown, as an exception to the theory of jura
regalia.
MAIN POINT:
There is an existence of native title to land, or ownership of land by Filipinos by virtue of
possession under a claim of ownership since time immemorial and independent of any
grant from the Spanish Crown, as an exception to the theory of jura regalia.

CASE NO. 1234 - LAKBAO


ARTICLE XII: SECTION 2 - PUBLIC DOMAIN AND REGALIAN DOCTRINE
LAUREL V. GARCIA

FACTS: The subject Roppongi property is one of the properties acquired by the
Philippines from Japan pursuant to a Reparations Agreement. The property is where the
Philippine Embassy was once located, before it transferred to the Nampeidai property.
It was decided that the properties would be available to sale or disposition. One
of the first properties opened up for public auction was the Roppongi property, despite
numerous oppositions from different sectors.

ISSUE: (1) Can the Roppongi property and others of its kind be alienated by the
Philippine Government?; (2) Does the Chief Executive, her officers and agents, have the
authority and jurisdiction, to sell the Roppongi property?

RULING: The Roppongi property was acquired together with the other properties
through reparation agreements. They were assigned to the government sector and
that the Roppongi property was specifically designated under the agreement to house
the Philippine embassy. It is of public dominion unless it is convincingly shown that
the property has become patrimonial. The respondents have failed to do so.As property
of public dominion, the Roppongi lot is outside the commerce of man. It cannot be
alienated. Its ownership is a special collective ownership for general use and
payment, in application to the satisfaction of collective needs, and resides in the social
group. The purpose is not to serve the State as the juridical person but the citizens; it
is intended for the common and public welfare and cannot be the object of appropriation.
The fact that the Roppongi site has not been used for a long time for actual
Embassy service doesn’t automatically convert it to patrimonial property. Any such
conversion happens only if the property is withdrawn from public use. A property
continues to be part of the public domain, not available for
private appropriation or ownership until there is a formal declaration on the part of the
government to withdraw it from being such.

MP: The fact that the Roppongi site has not been used for a long time for actual Embassy
service does not automatically convert it to patrimonial property. Any such conversion
happens only if the property is withdrawn from public use. A property continues to be part
of the public domain, not available for private appropriation or ownership "until there is a
formal declaration on the part... of the government to withdraw it from being such.

CASE NO.1235 - SABTALUH


ARTICLE XII: PUBLIC DOMAIN AND REGALIAN DOCTRINE
ALMEDA VS. COURT OF APPEALS

FACTS: The case involves a parcel of land with an area of 1,208 square meters located
in Barrio Pampangin, Pateros, Rizal, and described in Survey Plan Psu-128539. The lot
was originally owned by the petitioners’ parents. On September 12, 1984, the petitioners,
filed for the registration of the land in the RTC of Pasig. The only opposition of the
application was filed by the Director of Lands through the Solicitor General on the
contention that the applicants have not met the statutory requirement on possession
under Section 48(b) of CA 141, mainly because the land applied for was alienable
forestland before its release as alienable and disposable land on January 3, 1968. The
applicants’ possession prior to January 3, 1968 was invalid for the purposes of a grant
under Section 48(b) of the Public Land Act, which states that actual possession should
be 30 years.

ISSUE: Whether or not classification of the land affects the vested rights of applicants
and their predecessor-in-interest who had exclusive and peaceful possession of said
parcel of land since 1918?

RULING: The Supreme Court upheld the decision of the CA, that the respondent had not
qualified for a grant under the Public Land Act because their possession of the land
while it was still inalienable forest land, or before it was declared alienable and
disposable land of the public domain on January 13, 1968, could not ripen into private
ownership, and should be excluded from the computation of the 30-year open and
continuous possession in concept of owner required under Section 48(b) of Com. Act141.

MAINPOINT: The thirty-year period only begins to toll only from the time the land is
converted into alienable public land.

CASE NO: 1236 - VARGAS


ARTICLE XII: SECTION 2 - PUBLIC DOMAIN AND REGALIAN DOCTRINE
DIRECTOR OF LANDS V. KALAHI INVESTMENTS

FACTS: An appeal from the decision of the Court of First Instance (now Regional Trial
Court) of Pampanga, denying the application of Kalahi Investments, Inc. for registration
of Lot No. 1851-B of the Floridablanca Cadastre, certified to this Court by the Court of
Appeals on January 11, 1978.

Kalahi abandoned its former claim over the entire area of Lot No. 1851-B, covering an
area of 886,021,588 square meters (Psd2387-D). It limited its present claim to 1,730
hectares, known as Lot No. 1 of Plan Sgs-3690, a portion of Lot 2210, Florida blanca
Cadastre, formerly a part of Lot No. 1851-B. It is in this Lot No. 1 of Plan Sgs-3690, with
an area of 1.284.2340 hectares, and in Lot No. 2, with an area of 446.0870 hectares,
giving a total of 1,720 hectares where the alleged 123 lode mining claims are said to be
existing and where the alleged 500,000 coffee plants were planted.

Kalahi presented evidence to support perfected mining rights over the 123 mineral
claims. These were, however, not considered by the court a quo as basis sufficient in law
and in fact for the registration of title under act 496.

ISSUE: Whether mining claims, acquired, registered, perfected, and patentable under
the Old Mining Law, mature to private ownership which would entitle the claimant-
applicant to the ownership thereof?

RULING: NO. The right of a locator of a mining claim is a property right, “this
right is not absolute. Mere location does not mean absolute ownership over the affected
land or the mining claim. It merely segregates the located land or area from the public
domain by barring other would-be locators from locating the same and appropriating for
themselves the minerals found therein. To rule otherwise would imply that location is all
that is needed to acquire and maintain rights over a located mining claim. This, we cannot
approve or sanction because it is contrary to the intention of the lawmaker that the locator
should faithfully and consistently comply with the requirements for annual work and
improvements in the located mining claims.

MAIN POINT: Bureau of Mines is qualified to rule on whether there has been full
and faithful compliance with the requirements of the Philippine Bill of 1902 as amended.

CASE NO. 1237 - TIPOE


ART. XII SEC. 2: REGALIAN DOCTRINE; PUBLIC DOMAIN
DIRECTOR OF LANDS v. CA [G.R. No. 112567. February 7, 2000.]

FACTS: Private respondent Aquilino Carino filed a petition for registration of a sugar land
in Laguna based on a declaration that such land belonged to his mother and by virtue of
an extra-judicial settlement, sole ownership of the land was granted him in 1949.

ISSUE: W/O respondent can claim ownership of the land based solely on his declaration.

RULING: NO. In order for a petition for registration may prosper, he must show proof that
he and his predecessor-in-interest had adverse possession and occupation of said land
for atleast 30 years which he failed to prove. Respondent has not submitted proof of his
fee simple title or proof of possession in the manner and for the length of time required
by the law to justify confirmation of an imperfect title.

MAIN POINT: Basic is the rule that the petitioner in a land registration case must prove
the facts and circumstances evidencing his alleged ownership of the land applied for.
General statements, which are mere conclusions of law and not factual proof of
possession are unavailing and cannot suffice.

CASE NO. 1238 - CHIONG


ARTICLE XII:SEC. 2 - PUBLIC DOMAIN AND REGALIAN DOCTRINE
REPUBLIC V. DE GUZMAN

FACTS:

After public bidding held on March 18, 1950, the Board of Liquidators,awarded Lot 5249
Ts-217, a 450 square meter land situated in Dadiangas, General Santos City, to Eusebio
Diones of Takurong, Bubon, Cotabato. On March 11, 1955, Eusebio Diono transferred
his rights over the lot to Enrique P. de Guzman (de Guzman, for brevity) for P700.00,
evidenced by an Agreement of Transfer of Right. On November 12, 1956, the Board of
Liquidators cancelled the award previously given to Eusebio Diones. On August 29, 1967,
the Director of Lands ordered the award and issuance of a patent in favor of de Guzman.
On September 5, 1967, the Department of Agriculture and Natural Resources approved
and issued Miscellaneous Sales Patent No. 814 to de Guzman. On September 26, 1967,
the Register of Deeds, General Santos, issued Original Certificate of Title No. P-29712 in
the name of de Guzman. On January 14, 1981, petitioner Republic of the Philippines filed
with the Court of First Instance, 16th Judicial District, General Santos City re-amended
complaint for reversion of Lot 5249 Ts-217 and cancellation of titles against Enrique P. de
Guzman, spouses Rio Rivera and Carolina R. de Guzman, the City Registrar of Deeds,
General Santos City and Hoechst Phils., Inc.

ISSUE:

Whether or not the Director of Lands loses authority over the land the moment an original
certificate of title is issued covering the same

RULING:

The authority of the Director of Lands to investigate conflicts over public lands is derived
from Section 91 of the Public Land Act. In fact, it is not merely his right but his specific
duty to conduct investigations of alleged fraud in securing patents and the corresponding
titles thereto. While title issued on the basis of a patent is as indefeasible as one judicially
secured, such indefeasibility is not a bar to an investigation by the Director of Lands as
to how such title had been acquired, if the purpose of such investigation is to determine
whether or not fraud had been committed in securing such title, in order that the
appropriate action for reversion may be filed by the Government.

MAIN POINT:

Actual possession of the land by the applicant and making improvements thereto were
among the legal requirements to be complied with by an applicant.
CASE NO. 1239 - LAKBAO
Article XII. National Economy and Patrimony. Section 2. Regalian Doctrine
Public Domain and Regalian Doctrine
Pua v. CA
Facts:
Issue:
Ruling:
MP:

CASE NO. 1240 - SABTALUH


ARTICLE XII: PUBLIC DOMAIN AND REGALIAN DOCTRINE
CRUZ VS. SECRETARY OF DENR
FACTS: Cruz assailed the validity of the RA8371 or the IndigenousPeople’s Rights Act
on the ground that the law amount to an unlawful deprivation of the State’s ownership
over lands of the public domain as well as minerals and other natural resources therein,
in violation of the regalian doctrine embodied in Section 2, ArticleXII of the Constitution.
The IPRA law basically enumerates the rights of the indigenous peoples over ancestral
domains which may include natural resources. Cruz et al content that, by providing for an
all-encompassing definition of “ancestral domains” and “ancestral lands” which might
even include private lands found within said areas, Sections 3(a) and 3(b) of said law
violate the rights of private landowners.

ISSUE: Whether or not the IPRA law is unconstitutional.

HELD: The SC deliberated upon the matter. After deliberation they voted and reached a
7-7 vote. They deliberated again and the same result transpired. Since there was no
majority vote, Cruz’s petition was dismissed and the IPRA law was sustained. Hence,
ancestral domains may include natural resources – somehow against the regalian
doctrine

MAINPOINT: Ancestral domain and ancestral lands are not part of lands of the public
domain, they are private lands belonging to indigenous people.

CASE NO: 1241 - VARGAS


ARTICLE XII; SECTION 2- PUBLIC DOMAIN AND REGALIAN DOCTRINE
CHAVEZ V. PEA

FACTS: President Marcos through a presidential decree created PEA, which was
tasked with the development, improvement, and acquisition, lease, and sale of all
kinds of lands. The then president also transferred to PEA the foreshore and offshore
lands of Manila Bay under the Manila-Cavite Coastal Road and Reclamation Project.
Thereafter, PEA was granted patent to the reclaimed areas of land and then, years
later, PEA entered into a JVA with AMARI for the development of the Freedom Islands.
These two entered into a joint venture in the absence of any public bidding.
Later, a privilege speech was given by Senator President Maceda denouncing
the JVA as the grandmother of all scams. An investigation was conducted and it was
concluded that the lands that PEA was conveying to AMARI were lands of the public
domain; the certificates of title over the Freedom Islands were void; and the JVA itself
was illegal. This prompted Ramos to form an investigatory committee on the legality of
the JVA. Petitioner now comes and contends that the government stands to lose
billions by the conveyance or sale of the reclaimed areas to AMARI. He also
asked for the full disclosure of the renegotiations happening between the parties.

ISSUE: Whether stipulations in the amended JVA for the transfer to AMARI of the
lands, reclaimed or to be reclaimed, violate the Constitution.

RULING: The ownership of lands reclaimed from foreshore and submerged areas is
rooted in the Regalian doctrine, which holds that the State owns all lands and waters of
the public domain. The 1987 Constitution recognizes the Regalian doctrine. It declares
that all natural resources are owned by the State and except for alienable agricultural
lands of the public domain, natural resources cannot be alienated. The Amended
JVA covers a reclamation area of 750 hectares. Only 157.84 hectares of the 750 hectare
reclamation project have been reclaimed, and the rest of the area are still submerged
areas forming part of Manila Bay. Further, it is provided that AMARI will reimburse
the actual costs in reclaiming the areas of land and it will shoulder the other reclamation
costs to be incurred.

MAIN POINT: The foreshore and submerged areas of Manila Bay are part of the lands of
the public domain, waters and other natural resources and consequently owned by the
State. As such, foreshore and submerged areas shall not be alienable unless they are
classified as agricultural lands of the public domain. The mere reclamation of these
areas by the PEA doesn’t convert these inalienable natural resources of the State
into alienable and disposable lands of the public domain. There must be a law or
presidential
proclamation officially classifying these reclaimed lands as alienable and disposable
if the law has reserved them for some public or quasi-public use.

CASE NO. 1242 - TIPOE


ART. XII SEC. 2: REGALIAN DOCTRINE; PUBLIC DOMAIN
DIDIPIO (DESAMA, INC.) V. GOZUN [G.R. NO. 157882. MARCH 30, 2006.]

FACTS: Republic Act No. 7942 or Philippine Mining Act of 1995, together with the IRR
issued pursuant thereto, Department of Environment and Natural Resources (DENR)
Administrative Order No. 96-40, s. 1996 (DAO 96-40) and of the Financial and Technical
Assistance Agreement (FTAA) entered into on 20 June 1994 by the Republic of the
Philippines. The foreign party consolidated with Climax Mining Limited to form a single
company that now goes under the new name of Climax-Arimco Mining Corporation
(CAMC), the controlling 99% of stockholders of which are Australian nationals.
ISSUE: W/N the state abdicated its primary responsibility to the full control and
supervision over natural resources in entering such agreement.

RULING: NO. The setup under RA 7942 and DAO 96-40 hardly relegates the State to
the role of a "passive regulator" dependent on submitted plans and reports. On the
contrary, the government agencies concerned are empowered to approve or disapprove
— hence, to influence, direct and change the various work programs and the
corresponding minimum expenditure commitments for each of the exploration,
development and utilization phases of the mining enterprise.

MAIN POINT: the FTAA contractor is not free to do whatever it pleases and get away
with it; on the contrary, it will have to follow the government line if it wants to stay in the
enterprise. Ineluctably then, RA 7942 and DAO 96-40 vest in the government more than
a sufficient degree of control and supervision over the conduct of mining operations.

CASE NO. 1243 - CHIONG


ARTICLE XII: SECTION 2. PUBLIC DOMAIN AND REGALIAN DOCTRINE
CHAVEZ V. NHA (2007)

FACTS:

On March 19, 1993, the National Housing Authority (NHA) and R-II Builders, Inc.
(RBI)entered into a Joint Venture Agreement (JVA) for the development of the Smokey
Mountain dumpsite and reclamation area to be converted into a low cost medium rise
housing complex and industrial/commercial site. The Project will involve 79 hectares of
reclaimed land (it was initially 40hectares but the JVA was amended). The JVA also
provides that as part of the consideration for the Project, NHA will convey a portion of the
reclaimed lands to RBI. the reclamation of the area was made; and subsequently, Special
Patents were issued conveying the reclaimed land to NHA. On August 5, 2004, former
Solicitor General Francisco I. Chavez filed this Petition for Prohibition and Mandamus
seeking to declare NULL and VOID the Joint Venture Agreement (JVA) and the Smokey
Mountain Development and Reclamation Project, and all other agreements in relation
thereto, for being Unconstitutional and Invalid.

ISSUE:

W/N NHA and RBI have been granted the power and authority to reclaim lands of the
public domain

RULING:

Yes. Although PEA was designated under EO 525 as the agency primarily responsible
for integrating, directing, and coordinating all reclamation projects, its charter does not
mention that it has the exclusive and sole power and authority to reclaim lands of public
domain. In fact, EO525 provides that reclamation projects may also be undertaken by a
national government agency or entity authorized by its charter to reclaim land.

MAIN POINT:

To lands reclaimed by PEA or through a contract with a private person or entity, such
reclaimed lands still remain alienable lands of public domain which can be transferred
only to Filipino citizens but not to a private corporation. This is because PEA under PD
1084 and EO 525 is tasked to hold and dispose of alienable lands of public domain and
it is only when it is transferred to Filipino citizens that it becomes patrimonial property

CASE NO. 1244 - LAKBAO


ARTICLE XII: SECTION 2. PUBLIC DOMAIN AND REGALIAN DOCTRINE
REPUBLIC V. ENCISO

FACTS: The respondent, alleging to be the owner in fee simple of a parcel of residential
land located in Barangay South Poblacion, Masinloc, Zambales, filed a petition for land
registration before the RTC of Iba, Zambales. The respondent averred, inter alia, that he
acquired title to the said lot by virtue of an extrajudicial settlement of estate and quitclaim
on March 15, 1999; the said property is not tenanted or occupied by any person other
than the respondent and his family who are in actual physical possession of the same;
and the respondent and his predecessors-in-interest have been in continuous, peaceful,
open, notorious, uninterrupted and adverse possession of the land in the concept of an
owner for not less than 30 years immediately preceding the filing of the application.
Petitioner Republic of the Philippines, through the Office of the Solicitor General (OSG),
opposed the application on the following grounds: (a) neither the respondent nor his
predecessors-in-interest have been in open, continuous, exclusive, and notorious
possession and occupation of the subject land since June 12, 1945 or prior thereto; (b)
the respondent failed to adduce any muniment of title and/or the tax declaration with the
application to prove bona fide acquisition of the land applied for or its open, continuous,
exclusive and notorious possession and occupation thereof in the concept of owner since
June 12, 1945 or prior thereto; (c) the alleged tax declaration adverted to in the application
does not appear to be genuine and the tax declarations indicate such possession to be
of recent vintage; (d) the claim of ownership in fee simple on the basis of Spanish title or
grant can no longer be availed of by the respondent considering that he failed to file an
appropriate application for registration within the period of six months from February 16,
1976 as required by P.D. No. 892; and (e) the subject land is a portion of the public
domain belonging to the Republic of the Philippines which is not subject to private
appropriation. The respondent presented tax receipts to show that the property was
declared for taxation purposes in his name. He also testified that he acquired the property
by inheritance from his deceased father, Vicente Enciso, who died on May 18, 1991. He
then immediately took possession of the property and constructed a house thereon in
1991. On March 15, 1999, he and his siblings executed an extrajudicial settlement of
estate where the land was adjudicated in his favor. The respondent further narrated that
the property was originally owned by the Municipality of Masinloc, Zambales. On October
5, 1968, the municipality passed Resolution No. 71,8 undertaking to construct a road
along the shoreline of the poblacion, but requiring landowners adjoining the roads to
share in the expenses for an inner wall adjacent to their lots.

ISSUE: Whether the subject land in this case is a residential land of private ownership or
still a land of public domain.

RULING: Subject land was reclassified as residential. It was already segregated from the
public domain and assumed the character of private ownership. It was reclaimed by the
Municipality of Masinloc and eventually adjudicated to Honorato Edaño. The Municipality
of Masinloc must have been in possession of the subject land even before 1969
considering that it was originally surveyed way back in 1927-1928. In the exercise of its
proprietary right, the Municipality of Masinloc validly conveyed the subject land to
petitioner-appellee’s predecessors-in-interest. Petitioner-appellee’s possession and
occupation of the subject land is continuous, public, adverse and uninterrupted and in the
concept an owner and no other person claimed possession and ownership of the same.
Indeed, there is nothing to support the respondent’s claim that the property "was
reclassified as residential … already segregated from the public domain and assumed
the character of private ownership." At the moment, it is not clear as to when the proper
authorities classified the subject as alienable and disposable. It must be stressed that
incontrovertible evidence must be presented to establish that the land subject of the
application is alienable or disposable.

MP: Properties of political subdivision[s] which are patrimonial in character may be


alienated. By analogy, when a municipality’s properties for public use are no longer
intended for such use, the same become patrimonial and may be the subject of a contract.
Thus, the Deed of Absolute Sale executed by and between the Municipal Mayor of
Masinloc and Honorato Edaño was a valid contract.

CASE NO. 1245 - SABTALUH


ARTICLE XII: PUBLIC DOMAIN AND REGALIAN DOCTRINE
PHILIPPINE GEOTHERMAL VS. NAPOCOR

FACTS: On September 10, 1971, the National Power Corporation (NPC) entered into a
service contract with Philippine Geothermal, Inc. (PGI), a corporation organized and
existing under the laws of California, United States of America, for the exploration and
exploitation of geothermal resources covering the Tiwi and Mak-Ban Geothermal Fields.
Section 3.1 of said contract provides: The term of this contract shall be twenty-five (25)
years renewable for another twenty-five (25) years upon the option of PGI under the same
terms and conditions set forth herein. Albeit the service contract was to expire in 1996,
the negotiations for its renewal started as early as 1994. NPC, however, was doubtful
whether a renewal would be constitutional in light of Section 2, Article XII of the 1987
Constitution, the NPC filed before the Regional Trial Court (RTC) of Quezon City a petition
for declaratory relief against PGI praying for the determination of the constitutionality of
Section 3 of the service contract. Pending decision the PGI and the NPC filed several
joint motions to suspend proceedings upon the ground that they were negotiating for the
settlement of the case. PGI and NPC filed a Joint Motion to Approve Compromise
Agreement and to Dismiss the earlier petition based on Compromise Agreement alleging
that, among others things: The Compromise Agreement is not contrary to law because it
in fact directly addressed to the very heart of the constitutional issues involved in this
controversy.

ISSUE: Whether or not the compromise agreement is constitutional.


RULING: NO. The Compromise Agreement is not contrary to public policy. It has been
categorically declared by the state that private sector participation and privatization of
state-owned enterprises and their assets is encouraged in order to accelerate economic
progress and development as evidenced by various laws and issuances Thus [PGI] and
[NPC] have agreed to terminate the Service Contract subject matter of the dispute, in
favor of a new Geothermal Sales Contract and a PD 1442 Geothermal Service Contract,
and PGI has committed to form a Philippine company for the development and operation
of the Tiwi and Mak-Ban steamfields (Sec. 6.1 thereof) on a going-forward basis, thereby
effectively erasing any doubt as to the legality of the compromise

MAINPOINT: The Compromise Agreement is not contrary to law because it in fact directly
addressed to the very heart of the constitutional issues involved in this controversy.
CASE NO: 1246 - VARGAS
ARTICLE XII: SECTION 2 - PUBLIC DOMAIN AND REGALIAN DOCTRINE
LA BUGAL-B’LAAN V. RAMOS

FACTS: The Petition for Prohibition and Mandamus before the Court challenges the
constitutionality of (1) Republic Act 7942 (The Philippine Mining Act of 1995); (2) its
Implementing Rules and Regulations (DENR Administrative Order [DAO] 96-40); and (3)
the Financial and Technical Assistance Agreement (FTAA) dated 30 March 1995,
executed by the government with Western Mining Corporation (Philippines), Inc.
(WMCP).
On 27 January 2004, the Court en banc promulgated its Decision, granting the Petition
and declaring the unconstitutionality of certain provisions of RA 7942, DAO 96-40, as well
as of the entire FTAA executed between the government and WMCP, mainly on the
finding that FTAAs are service contracts prohibited by the 1987 Constitution. The
Decision struck down the subject FTAA for being similar to service contracts,[9] which,
though permitted under the 1973 Constitution, were subsequently denounced for being
antithetical to the principle of sovereignty over our natural resources, because they
allowed foreign control over the exploitation of our natural resources, to the prejudice of
the Filipino nation.
The Decision quoted several legal scholars and authors who had criticized service
contracts for, inter alia, vesting in the foreign contractor exclusive management and
control of the enterprise, including operation of the field in the event petroleum was
discovered; control of production, expansion and development; nearly unfettered control
over the disposition and sale of the products discovered/extracted; effective ownership of
the natural resource at the point of extraction; and beneficial ownership of our economic
resources. According to the Decision, the 1987 Constitution (Section 2 of Article XII)
effectively banned such service contracts. Subsequently, Victor O. Ramos (Secretary,
DENR), Horacio Ramos (Director, Mines and Geosciences Bureau [MGB-DENR]), Ruben
Torres (Executive Secretary), and the WMC (Philippines) Inc. filed separate Motions for
Reconsideration.

ISSUE: Whether the Court has a role in the exercise of the power of control over the EDU
of our natural resources?

RULING: The Chief Executive is the official constitutionally mandated to “enter into
agreements with foreign owned corporations.” On the other hand, Congress may review
the action of the President once it is notified of “every contract entered into in accordance
with this [constitutional] provision within thirty days from its execution.” In contrast to this
express mandate of the President and Congress in the exploration, development and
utilization (EDU) of natural resources, Article XII of the Constitution is silent on the role of
the judiciary. However, should the President and/or Congress gravely abuse their
discretion in this regard, the courts may -- in a proper case -- exercise their residual duty
under Article VIII. Clearly then, the judiciary should not inordinately interfere in the
exercise of this presidential power of control over the EDU of our natural resources.

MAIN POINT: Constitution should be read in broad, life-giving strokes. It should not be
used to strangulate economic growth or to serve narrow, parochial interests. Rather, it
should be construed to grant the President and Congress sufficient discretion and
reasonable leeway to enable them to attract foreign investments and expertise, as well
as to secure for our people and our posterity the blessings of prosperity The and peace.

CASE NO. 1247 - TIPOE


ART. XII SEC. 2: REGALIAN DOCTRINE; PUBLIC DOMAIN
J.G. SUMMIT HOLDINGS, INC. V. CA [G.R. NO. 124293. JANUARY 31, 2005.]

Facts: Investment and Development Corporation (NIDC) and KAWASAKI of Japan


entered into a Joint Venture Agreement (JVA) for the construction, operation, and
management of Subic National Shipyard, Inc. which eventually became the Philippine
Shipyard and Engineering Corporation (PHILSECO). Under the JVA, KAWASAKI will
contribute and obtain 40% ownership granted right of first refusal should either party
decide to transfer interest in the corporation.

Issue: W/N KAWASAKI, a foreign corporation, can possess more than 40% interest in
PHILSECO.

Ruling: YES. No law disqualifies a person from purchasing shares in a landholding


corporation even in excess of the allowed foreign equity.
Main Point: Art. XII Sec. 2 of the Constitution states that the utilization of natural
resources shall be under the full control and supervision of the State. Philippines may
enter into joint venture with corporations at least 60% of whose capital is owned by such
citizens. It contemplates disqualification from owning land and not from foreign ownership
of shares.

CASE NO. 1248- CHIONG


ARTICLE XII: SECTION 2. - ALIENATION
SANTA ROSA MINING COMPANY VS LEIDO

FACTS:

Presidential decree no. 1214 issued requiring holders of subsisting and valid patentable
mining claims located under the provisions of the Philippine bill of 1902 to file a mining
lease of application within one (1) year from the approval of the decree. To protect its
rights, petitioner santa rosa mining company files a civil action for certiorari and prohibition
confronting the said decree as unconstitutional in that it amounts to a deprivation of
property without due process of law. Subsequently, three(3) days after, petitioner filed a
mining lease application, but “under protest”, with a reservation that it is not waiving its
rights over mining claims until the validity of the decree shall have been passed upon by
the court. The respondents alleged that petitioner has no standing to file the instant
petition and question the decree as it failed to fully exhaust administrative remedies.

ISSUE:

Won Presidential Decree No. 1214 is constitutional

RULING:

Yes, Presidential Decree No. 1214 is constitutional, even assuming arguendo that
petitioners was not bound to exhaust administrative remedies for its mining claims to be
valid in the outset. It is a valid exercise of the sovereign power of the state, as owner,
over the lands of the public domain, of which petitioner’s mining claims still form a part.
Moreover, Presidential Decree No. 1214 is in accord with Sec. 8, Art XIV of the 1937
constitution.

MAIN POINT:

It is a valid exercise of the sovereign power of the State, as owner, over lands of the public
domain, of which filing a mining lease application within one (1) year from the approval of
the Decree, and over the patrimony of the nation, of which mineral deposits are a valuable
asset.
CASE NO. 1249 - LAKBAO
ARTICLE XII:SECTION 2. REGALIAN DOCTRINEALIENATION
SAN MIGUEL CORP. V. CA

FACTS: San Miguel Corporation (SMC for brevity) purchased from Silverio Perez Lot
684, a 14.531-square-meter parcel of land located in Sta. Anastacia, Sto. Tomas,
Batangas, in consideration of the sum of P133, 084.80.[2] On February 21, 1977,
claiming ownership in fee simple of the land, SMC filed before the then Court of First
Instance, now Regional Trial Court of Batangas an application for its registration under
the Land Registration Act. The Solicitor General, appearing for the Republic of the
Philippines, opposed the application for registration contending that SMC's claim of
ownership in fee simple on the basis of a Spanish title or grant could no longer be availed
of by the applicant as the six-month period from February 16, 1976 prescribed by
Presidential Decree No. 892 had elapsed; that the parcel of land in question is part of the
public domain, and that SMC, being a private corporation, is disqualified under Section
11, Article XIV of the Constitution from holding alienable lands of the public domain. The
Solicitor General thereafter authorized the Provincial Fiscal of Batangas to appear in said
case, subject to his supervision and control.

Issue: Whether Republic of the Philippines can opposed the application for registration
of land by SMC and the subject parcel of land in question is part of the public domain.

Ruling: Yes. In this case, petitioner's claim that its predecessor-in-interest had open,
exclusive and undisputed possession of Lot 684 for more than thirty years is anchored on
certain documentary and testimonial evidence. Its documentary evidence consist of tax
declaration No. 923 wherein it appears that in 1974, Silverio Perez declared as his own
for taxation purposes, a certain Riceland with an area of 1.5657 hectares located in Sta.
Anastacia, Sto. Tomas, Batangas,and a certification of the Office of the Treasurer of Sto.
Tomas to the effect that in 1977, Silverio Perez paid realty taxes for the land subject of
tax declaration no. 923. Tax declarations and receipts are not conclusive evidence of
ownership or right of possession over a piece of land. They are merely indicia of a claim
of ownership. Tax declarations only become strong evidence of ownership of land
acquired by prescription, a mode of acquisition of ownership relied upon by petitioner in
this case, when accompanied by proof of actual possession. Such proof of actual
possession was sought to be provided by the testimony of vendor Silverio Perez that he
had been in possession of the property since 1933 until he sold it to SMC in 1975; that
the property was given to him by his parents when he got married; that no document
evidenced that transfer; that it had been in the possession of his parents since 1925; that
he had declared the property in his name for taxation purposes; that he had paid taxes
therefor, and that he was in peaceful, continuous and exclusive possession of the
property until its sale to SMC. We hold that there is paucity of evidence of actual, notorious
and exclusive possession of the property on the part of vendor Silverio Perez so as to
attach to it the character of an express grant from the govemment. Indeed, as correctly
held by the Court of Appeals, Silverio Perez's testimony, being uncorroborated, is simply
self-serving and hence, undeserving of any weight
MP: Open, exclusive and undisputed possession of alienable public land for the period
prescribed by law creates the legal fiction whereby the land, upon completion of the
requisite period ipso, jure and without the need of judicial or other sanction, ceases to be
public land and becomes private property. Such open, continuous, exclusive and
notorious occupation of the disputed properties for more than 30 years must, however,
be conclusively established. This quantum of proof is necessary to avoid the erroneous
validation of actually fictitious claims of possession over the property in dispute.

CASE NO. 1250 - SABTALUH


ARTICLE XII: ALIENATION
REPUBLIC VS BANTIGUE POINT DEVELOPMENT CORPORATION

FACTS:
Respondent Bantigue point development corporation filed with the RTC of Rosario,
Batangas an application for original registration of title over a parcel of land located at
barangay Barualte, San Juan, Batangas. On 18 july 1997, the rtc issued an order setting
the case for initial hearing on 22 and issued a second order setting the initial hearing on
4 November 1997. Petitioner republic filed its opposition to the application for registration
on 8 January 1998 while the records were still with the RTC.on 31 march 1998, the rtc
clerk of court transmitted motu proprio the records of the case to the MTC Of San Juan,
because the assessed value of the property was allegedly less than ₱100,000.
Thereafter, the MTC awarded the land to respondent corporation. The ca further found
that respondent corporation had sufficiently established the latter’s registrable title over
the subject property after having proven open, continuous, exclusive and notorious
possession and occupation of the subject land by itself and its predecessors-in-interest
even before the outbreak of world war ii dissatisfied with the ca’s ruling, petitioner republic
filed this instant petition and raised the arguments that municipal trial court failed to
acquire jurisdiction over the application for original registration of land title.

ISSUE: Whether or Not the Municipal Trial Court Has Jurisdiction Over the Application
For Original Registration Of Land Title

RULING: YES. The municipal trial court has jurisdiction over the application for original
registration of land title, Sec. 34. Delegated Jurisdiction in Cadastral and Land
Registration Cases. - Metropolitan Trial Courts, Municipal Trial Courts, and Municipal
Circuit Trial Courts may be assigned by the Supreme Court to hear and determine
cadastral or land registration cases covering lots where there is no controversy or
opposition, or contested lots where the value of which does not exceed One hundred
thousand pesos (₱100,000.00), such value to be ascertained by the affidavit of the
claimant or by agreement of the respective claimants if there are more than one, or from
the corresponding tax declaration of the real property. Their decision in these cases shall
be appealable in the same manner as decisions of the Regional Trial Courts.

MAINPOINT: Section 34 of the Judiciary Reorganization Act provides that the value of
the property sought to be registered may be ascertained in three ways: first, by the
affidavit of the claimant; second, by agreement of the respective claimants, if there are
more than one; or, third, from the corresponding tax declaration of the real property

CASE NO: 1251 - VARGAS


ARTICLE XII: SECTION 2- REGALIAN DOCTRINE; UTILIZATION
MINERS V FACTORAN

FACTS: Former President Corazon Aquino issued Executive Order Nos 211 and 279 in
the exercise of her legislative powers. EO No. 211 prescribes the interim procedures in
the processing and approval of applications for the exploration, development and
utilization of minerals pursuant to Section 2, Article XII of the 1987 Constitution. EO No.
279 authorizes the DENR Secretary to negotiate and conclude joint-venture, co-
production, or production- sharing agreements for the exploration, development, and
utilization of mineral resources.
The issuance and the impeding implementation by the DENR of Administrative Order
Nos. 57 which declares that all existing mining leases or agreements which were granted
after the effectivity of the 1987 Constitution…shall be converted into production-sharing
agreements within one (1) year from the effectivity of these guidelines.” and
Administrative Order No. 82 which provides that a failure to submit Letter of Intent and
Mineral Production-Sharing Agreement within 2 years from the effectivity of the
Department Administrative Order No. 57 shall cause the abandonment of the mining,
quarry, and sand and gravel claims, after their respective effectivity dates compelled the
Miners Association of the Philippines, Inc., an organization composed of mining
prospectors and claim owners and claim holders, to file the instant petition assailing their
validity and constitutionality before this Court.

ISSUE: Whether the two Department Administrative Orders valid?

RULING: Yes. Petitioner's insistence on the application of Presidential Decree No. 463,
as amended, as the governing law on the acceptance and approval of declarations of
location and all other kinds of applications for the exploration, development, and utilization
of mineral resources pursuant to Executive Order No. 211, is erroneous. Presidential
Decree No. 463, as amended, pertains to the old system of exploration, development and
utilization of natural resources through "license, concession or lease" which, however,
has been disallowed by Article XII, Section 2 of the 1987 Constitution. By virtue of the
said constitutional mandate and its implementing law, Executive Order No. 279 which
superseded Executive Order No. 211, the provisions dealing on "license, concession or
lease" of mineral resources under Presidential Decree No. 463, as amended, and other
existing mining laws are deemed repealed and, therefore, ceased to operate as the
governing law.

MAIN POINT: Accordingly, the State, in the exercise of its police power in this regard,
may not be precluded by the constitutional restriction on non-impairment of contract from
altering, modifying and amending the mining leases or agreements granted under
Presidential Decree No. 463, as amended, pursuant to Executive Order No. 211. Police
Power, being co-extensive with the necessities of the case and the demands of public
interest; extends to all the vital public needs. The passage of Executive Order No. 279
which superseded Executive Order No. 211 provided legal basis for the DENR Secretary
to carry into effect the mandate of Article XII, Section 2 of the 1987 Constitution.

CASE NO. 1252 - TIPOE


ART. XII SEC. 2: REGALIAN DOCTRINE; UTILIZATION
TANO V. SOCRATES [G.R. NO. 110249. AUGUST 21, 1997.]

FACTS: The Sangguniang Panlalawigan of Palawan enacted a resolution and an


ordinance prohibiting the catching, gathering, possessing, buying, selling and shipment
of live marine coral dwelling aquatic organisms for a period of five years. Issuances were
supported under R.A. 7611, otherwise known as the Strategic Environment Plan (SEP)
for Palawan Act, which adopts a comprehensive framework for the sustainable
development of Palawan compatible with protecting and enhancing the natural resources
and endangered environment of the province.

ISSUE: W/N such issuances violate the constitutional preferential rights of subsistence
and marginal fishermen.

RULING: NO. The so-called "preferential right" of subsistence or marginal fishermen to


the use of marine resources is not at all absolute.

MAIN POINT: In accordance with the Regalian Doctrine, marine resources belong to the
State, and, pursuant to Section 2, Article XII of the Constitution, their "exploration,
development and utilization . . . shall be under the full control and supervision of the
State." Moreover, their mandated protection, development and conservation as
necessarily recognized by the framers of the Constitution, imply certain restrictions on
whatever right of enjoyment there may be in favor of anyone.

A marginal fisherman is an individual engaged in fishing whose margin of return or


reward in his harvest of fish as measured by existing price levels is barely sufficient to
yield a profit or cover the cost of gathering the fish, while a subsistence fisherman is
one whose catch yields but the irreducible minimum for his livelihood.
CASE NO. 1253 - CHIONG
ARTICLE XIIL: SECTION 2 - REGALIAN DOCTRINE UTILIZATION
VILLAFLOR V. CA

FACTS:

The controversy is concerning parcels of public land under contest between Vicente
Villaflor and Nasipit Lumber. Their relationship began when Villaflor leased said parcels
to Nasipit (despite the investigator of lands reporting that he had no transmissible rights
over said parcels) before selling them to the latter. Said parcels, unfortunately, were still
public lands and both parties had to bid at a public option for a government grant to own
them. Villaflor was the preferred bidder and matched Nasipit’s bid. He, however, had
pressing matters to attend to and changed his residence to Manila, inducing him to creat
a deed relinquishing his rights in Nasipit’s favor in exchange for money. Consequently,
the Director of Lands awarded the parcels to the corporation. After ten years in Indonesia,
however, Villaflor wanted his land back. He died during the whole process by the way,
just thought you should know. He claimed: a. Lands were his private property.

b.He did not know of the grant: “Order of Award,” and the Bureau of Lands and all the
other courts ignored this.

c. Deed of Relinquishment was not paid for.

d. Corporations could not own land because of the 1973 Constitution.

ISSUE:

Whether or not the private respondent corporation is qualified to acquire ownership over
the land of public domain.

RULING:

Yes. Nasipit is qualified. Public land was acquired before 1973. Villaflor had already
relinquished his rights in favor of Nasipit. All told, the only disqualification that can be
imputed to private respondent is theprohibition in the 1973 Constitution against the
holding of alienable lands of the publicdomain by corporations. However, this Court
settled the matter, declaring that saidconstitutional prohibition had no retroactive effect
and could not prevail over a vested right to the land. Application for patent for and in
behalf of Nasipit has clearly no impediment, for they have proven satisfactory compliance
of the requirements of the law.

MAIN POINT:

It is well-settled that no public land can be acquired by private persons without any grant,
express or implied from the government. It is indispensable then that there be showing of
title from the state or any other mode of acquisition recognized by law. It is well-settled
that all lands remain part of the public domain unless severed there from by state grant
or unless alienated in accordance with law. The deeds of sale do not constitute clear and
convincing evidence to establish that the contested area is of private ownership. Hence,
the property must be held to be public domain

CASE NO. 1254 - LAKBAO


Article XII. National Economy and Patrimony. Section 2. Regalian Doctrine
Utilization
Republic v. CA and PREC

Facts:
Issue:
Ruling:
MP:

CASE NO. 1255 - SABTALUH


ARTICLE XII: UTILIZATION
REPUBLIC VS. ROSEMOOR REPUBLIC OF THE PHILIPPINES VS. ROSEMOOR
MINING AND DEVELOPMENT CORPORATION, ET AL. {G.R. NO. 149927 MARCH
30, 2004}

FACTS: Petitioner Rosemoor Mining and Development Corporation after having been
granted permission to prospect for marble deposits in the mountains of Biak-na-Bato, San
Miguel, Bulacan, succeeded in discovering marble deposits of high quality and in
commercial quantities in Mount Mabio which forms part of the Biak-na-Batomountain
range. The petitioner then applied with the Bureau of Mines, now Mines and Geosciences
Bureau, for the issuance of the corresponding license to exploit said marble deposits.
License No. 33 was issued by the Bureau of Mines in favor of the herein petitioners.
Shortly thereafter, Respondent Ernesto Maceda cancelled the petitioner’s license stating
that their license had illegally been issued, because it violated Section 69 of PD 463; and
that there was no more public interest served by the continued existence or renewal of
the license. The latter reason was confirmed by the language of Proclamation No. 84.
According to this law, public interest would be served by reverting the parcel of land
that was excluded by Proclamation No. 2204 to the former status of that land as part of
the Biak-na-Bato national park. The respondents argue that Proclamation No. 84 is a
invalid for being an ex post facto law and a bill of attainder.

ISSUE: Whether or not Presidential Proclamation No. 84 is valid.


HELD: Yes. Proclamation No. 84 is a bill of attainder; that is, a legislative act which inflicts
punishment without judicial trial. Its declaration that QLP No. 33 is a patent nullity is
certainly not a declaration of guilt. Neither is the cancellation of the license a punishment
within the purview of the constitutional proscription against bills of attainder. Too, there is
no merit in the argument that the proclamation is an ex post facto law. It is settled that
an ex post facto law is limited in its scope only to matters criminal in nature. Proclamation
84, which merely restored the area excluded from the Biak-na-Bato national park
by canceling respondents’ license, is clearly not penal in character.

MAINPOINT: Declaration of patent nullity is certainly not a declaration of guilt.

CASE NO: 1256 - VARGAS


ARTICLE XII: SECTION 2 - REGALIAN DOCTRINE;UTILIZATION
ALVAREZ V PICOP

FACTS: PICOP filed with the DENR an application to have its Timber License Agreement
(TLA) No. 43converted into an IFMA.PICOP filed before the (RTC) City a Petition for
Mandamus against then DENR Sec Alvarez for unlawfully refusing and/or neglecting to
sign and execute the IFMA contract of PICOP even as the latter has complied with all the
legal requirements for the automatic conversion of TLA No. 43, as amended, into an
IFMA. The cause of action of PICOP Resources, Inc. (PICOP) in its Petition for
Mandamus with the trial court is clear: the government is bound by contract, a 1969
Document signed by then President Ferdinand Marcos, to enter into an Integrated Forest
Management Agreement (IFMA) with PICOP.

ISSUE: Whether the 1969 Document is a contract recognized under the non-impairment
clause by which the government may be bound (for the issuance of the IFMA)

RULING: NO. Our definitive ruling in Oposa v. Factoran that a timber license is not a
contract within the purview of the non-impairment clause is edifying. We declared:
Needless to say, all licenses may thus be revoked or rescinded by executive action. It is
not a contract, property or a property right protected by the due process clause of the
Constitution.

MAIN POINT: The Presidential Warranty cannot, in any manner, be construed as a


contractual undertaking assuring PICOP of exclusive possession and enjoyment of its
concession areas. Such an interpretation would result in the complete abdication by the
State in favor of PICOP of the sovereign power to control and supervise the exploration,
development and utilization of the natural resources in the area.

CASE NO. 1257 - TIPOE


ART. XII SEC. 2: REGALIAN DOCTRINE; UTILIZATION
INITIATIVES FOR DIALOGUE AND EMPOWERMENT THROUGH ALTERNATIVE
LEGAL SERVICES, INC. (IDEALS, INC.) V. POWER SECTOR ASSETS AND
LIABILITIES MANAGEMENT CORPORATION (PSALM) [G.R. NO. 192088.
OCTOBER 9, 2012.]

FACTS: Petitioners assert that the terms of the sale of Angat Hydro-Electric Power Plant
(AHEPP) allowing the buyer, Korea Water Resources Corporation (K-Water), the
operation and management of the Non-Power Components, constitutes a relinquishment
of government control over the Angat Dam, in violation of the constitution.

ISSUE: W/N such sale violates Art. XII, Sec. 2 of the Constitution on alienation of natural
resources.

RULING: NO. K-Water will be a mere operator of the Angat Dam. In the power generation
activity, K-Water will have to utilize the waters already extracted from the river and
impounded on the dam. This process of generating electric power from the dam water
entering the power plant thus does not constitute appropriation within the meaning of
natural resource utilization in the Constitution.

MAIN POINT: While both waters and geothermal steam are, undoubtedly "natural
resources", within the meaning of Section 2 Article XII of the present Constitution, hence,
their exploitation, development and utilization should be limited to Filipino citizens or
corporations or associations at least sixty per centum of the capital of which is owned by
Filipino citizens, the utilization thereof can be opened even to foreign nationals, after the
same have been extracted from the source by qualified persons or entities . The rationale
is because, since they no longer form part of the natural resources of the country, they
become subject to ordinary commerce.

CASE NO. 1258 - CHIONG (DIGEST TOO LONG)


ARTICLE XII: SECTION 3 - LANDS OF THE PUBLIC DOMAIN
DIRECTOR OF LANDS VS. AQUINO 192 SCRA 296

FACTS:
The center of controversy for the review of petition is a limestone-rich 70-hectare land in
Bucay, Abra 66 hectares of which are, according to petitioners, within the Central
Cordillera Forest Reserve. Private respondent Abra Industrial Corporation (AIC for
brevity), a duly registered corporation established for the purpose of setting up a cement
factory, claims on the other hand, to be the owner in fee simple of the whole 70-hectare
area indicated in survey plans PSU-217518, PSU-217519 and PSU-217520 with a total
assessed value of P6,724.48. Thus, on September 23, 1965, it filed in the then Court of
First Instance of Abra an application for registration in its name of said parcels of land
under the Land Registration Act or, in the alternative, under Sec. 48 of Commonwealth
Act No. 141 1 as amended by Republic Act No. 1942 inasmuch as its predecessors-in-
interest had allegedly been in possession thereof since July 26, 1894. The requisite
publication and posting of notice having been complied with, the application was set for
hearing. Except for the Director of Lands, nobody appeared to oppose the application.
Hence, the court issued an order of default against the whole world except the Director
of Lands. After the applicant had rested its case, the provincial fiscal, appearing for the
Director of Lands, submitted evidence supporting the opposition filed by the Solicitor
General to the effect that AIC had no registerable title and that the highly mineralized
parcels of land applied for were within the Central Cordillera Forest Reserve which had
not yet been released as alienable and disposable land pursuant to the Public Land Law.
On July 22, 1966, the lower court 3 favorably acted on the application and ordered the
registration of the parcels of land under the Land Registration Act. Then the Director of
Lands filed a motion for reconsideration. AIC having filed its opposition to the motion for
reconsideration, the lower court denied it on September 28, 1967 holding that the grounds
raised therein were relevant and proper only if the Bureau of Forestry and the Bureau of
Mines were parties to the case. AIC filed a motion to dismiss the instant petition on the
grounds that it raises "unsubstantial" issues and that it was filed out of time. The motion
was denied by the Court 13 but it bears pointing out that AIC's second ground for
dismissal, which is premised on its perception that a motion for reconsideration of the
order of November 27, 1969 is necessary before the filing of the instant petition, is
incorrect. A motion for new trial or reconsideration is not a prerequisite to an appeal,
petition for review or a petition for review on Certiorari. Petitioners herein contend that the
lower court erred in granting the application for registration of the parcels of land
notwithstanding its finding that they are within the forest zone. They also argued that the
lower court erred in denying the petition for review based on actual fraud because under
Section 38 of Act No. 496, a decree of registration may be reviewed not only by reason
of actual fraud but also for a fatal infirmity of the decision upon which the decree is based,
provided no innocent purchaser for value will be prejudiced.
ISSUE:
Whether or not the lower court erred in granting the application for registration of the
parcels of land notwithstanding its finding that they are within the forest zone.
RULING:
The Court found the petition to be meritorious. They reiterated the rule enunciated by this
Court in Director of Forestry vs. Muñoz 18 and consistently adhered to in a long line of
cases 19 the more recent of which is Republic vs. Court of Appeals, 20 that forest lands
or forest reserves are incapable of private appropriation and possession thereof, however
long, cannot convert them into private properties. This ruling is premised on the Regalian
doctrine enshrined not only in the 1935 and 1973 Constitutions but also in the 1987
Constitution Article XIII of which provides that:
"Sec. 2. All lands of the public domain, waters, minerals, coal . . . , forests or timber, . . .
and other natural resources are owned by the State. With the exception of agricultural
lands, all other natural resources shall not be alienated." The petitioners therefore validly
insisted on the review of the decision ordering the issuance of the decree of registration
in view of its patent infirmity.
MAIN POINT:
The classification of the land as forest land is descriptive of its legal nature or status and
does not have to be descriptive of what the land actually looks like. Moreover, only a
positive act by the chief executive is needed to change such. Thus, in the case at bar, the
respondent corporation has to prove that the lands it claims for registration are alienable
or disposable lands, which it failed to so

CASE NO. 1259 - LAKBAO


Article XII. National Economy and Patrimony. Section 3. Lands of the Public
Dommain
Republic v. CA Gr. 103882

Facts:
Issue:
Ruling:
MP:

CASE NO. 1260- SABTALUH


ARTICLE XII: SEC.3 – LANDS OF PUBLIC DOMAIN
APEX MINING VS. SOUTHEAST MINDANAO GOLD, INC,

FACTS:
SEM filed a motion for reconsideration on the Decision held that the assignment of
Exploration Permit (EP) 133 in favor of SEM violated one of the conditions stipulated in
the permit. It also ruled that the transfer of EP 133 violated Presidential Decree No. 463,
which requires that the assignment of a mining right be made with the prior approval of
the Secretary of the Department of Environment and Natural Resources (DENR).
Moreover, the Assailed Decision pointed out that EP 133 expired by non-renewal since it
was not renewed before or after its expiration. It likewise upheld the validity of
Proclamation No. 297 absent any question against its validity. In view of this, and
considering that under Section 5 of Republic Act No. 7942, otherwise known as the
“Mining Act of 1995,” mining operations in mineral re servations may be undertaken
directly by the State or through a contractor, Thus, it was held in the Assailed Decision
that it is now within the prerogative of the Executive Department to undertake directly the
mining operations of the disputed area or to award the operations to private entities
including petitioners Apex and Balite, subject to applicable laws, rules and regulations,
and provided that these private entities are qualified. SEM assailed the constitutionality
of Proclamation No. 297.

ISSUE: Whether or not the issuance of Proclamation No. 297 violates Section 4,
Article XII of the Constitution.

RULING: No, Proclamation No. 297 Is in Harmony with Article XII, Section 4, of the
Constitution. SEM asserts that Article XII, Section 4 of the Constitution, bars the President
from excluding forest reserves/reservations and proclaiming the same as mineral
reservations, since the power to de-classify them resides in Congress. there is nothing in
the constitutional provision that prohibits the President from declaring a forest land as an
environmentally critical area and from regulating the mining operations therein by
declaring it as a mineral reservationn order to prevent the further degradation of the forest
environment and to resolve the health and peace and order problems that beset the area,
the proclamation in question is in line with Section 4, Article XII of the Constitution, as the
former fosters the preservation of the forest environment of the Diwalwal area and is
aimed at preventing the further degradation of the same. These objectives are the very
same reasons why the subject constitutional provision is in place.

MAINPOINT: The president has been given the power to withdraw forest reserves found
to be more valuable for their mineral contents than for the purspose for which the
reservation was made and convert the same into non-forests reserves.

CASE NO: 1261 - VARGAS


ARTICLE XII: SECTION 2 -LANDS OF THE PUBLIC DOMAIN
DIR. OF LANDS V. IAC
FACTS: Acme Plywood & Veneer Co., Inc., a corp. represented by Mr. Rodolfo Nazario,
acquired from Mariano and Acer Infiel, members of the Dumagat tribe 5 parcels of land
possession of the Infiels over the land dates back before the Philippines was discovered
by Magellan land sought to be registered is a private land pursuant to RA 3872 granting
absolute ownership to members of the non-Christian Tribes on land occupied by them or
their ancestral lands, whether with the alienable or disposable public land or within the
public domain. Acme Plywood & Veneer Co. Inc., has introduced more than P45M worth
of improvements ownership and possession of the land sought to be registered was duly
recognized by the government when the Municipal Officials of Maconacon, Isabela
donated part of the land as the town site of Maconacon Isabela.

ISSUE: Whether the constitutional prohibition against their acquisition by private


corporations or associations applies

RULING: No. If it is accepted-as it must be-that the land was already private land to which
the Infiels had a legally sufficient and transferable title on October 29, 1962 when Acme
acquired it from said owners, it must also be conceded that Acme had a perfect right to
make such acquisition. The only limitation then extant was that corporations could not
acquire, hold or lease public agricultural lands in excess of 1,024 hectares.

MAIN POINT: Operation of law not only a right to a grant, but a grant of the Government,
for it is not necessary that a certificate of title should be issued in order that said grant
may be sanctioned by the courts, an application therefore is sufficient.

CASE NO. 1262 - TIPOE


ART. XII SEC. 3: LANDS OF THE PUBLIC DOMAIN
TEN FORTY REALTY AND DEVELOPMENT CORP. V. CRUZ [G.R. NO. 151212.
SEPTEMBER 10, 2003.]

FACTS: Galino allegedly sold a parcel of residential house and lot situated in Olongapo
City to petitioner in 1996, and then sold the same property to respondent in 1998.
Petitioner argued that being the first buyer, it has a better right to own the realty.

ISSUE: W/N petitioner, a corporation, can even possess such parcel of lot.
RULING: NO. Private corporations are disqualified from acquiring lands of the public
domain, as provided under Section 3 of Article XII of the Constitution.

MAIN POINT: While corporations cannot acquire land of the public domain, they can
however acquire private land. The established rule is that alienable and disposable land
of the public domain held and occupied by a possessor — personally or through
predecessors-in-interest, openly, continuously, and exclusively for 30 years — is ipso jure
converted to private property by the mere lapse of time. However, petitioner has not
presented proof that, at the time it purchased the property from Galino, the property had
ceased to be of the public domain and was already private land.

CASE NO. 1263 - CHIONG


ARTICLE XII: SECTION 3. LANDS OF THE PUBLIC DOMAIN
CHAVEZ VS PEA

FACTS:

Public Estates Authority (PEA) is a wholly government-owned and –controlled


corporation which is the primary implementing agency of the National Government to
reclaim foreshore and submerged lands of the public domain. By virtue of a special Patent
issued by President Corazon Aquino, the register of deeds of the Paranaque, in April
1988,issued certificates of title, in the name of PEA, covering three reclaimed islands
known as the freedom islands located at the southern portion of the Manila-Cavite coastal
road, Paranaque city. The freedom islands have a total land area of 157.841 hectares. In
April 1995, PEA entered into a Joint Venture Agreement (JVA) with AMARI, a private
corporation, to develop the freedom Islands. The JVA also required the reclamation of an
additional 728 hectares of submerged areas surrounding these islands to complete the
configuration in the Master Development Plan of the Southern reclamation Project-Manila
Cavite Coastal Road Reclamation Project . The JVA was later amended giving AMARI
an option to reclaim an additional 350 hectares of submerged areas. Part of the
consideration for AMARI’s work is the conveyance of 70% of the total net usable
reclaimed area – equivalent to 367.5 hectares, title of which will be in AMARI’s name.

ISSUE:

Whether or not AMARI, a private corporation, can acquire and own under the Amended
JVA 367.5 hectares of reclaimed foreshore and submerged areas in Manila Bay

RULING:
No. AMARI as a private corporation cannot acquire the reclaimed freedom Islands,
though alienable lands of the public domain, except by lease, as provided under Section
3, Article XII of the Constitution. The still submerged areas (i.e., the more or less
additional 250 and 350 hectares of submerged areas) in Manila Bay are inalienable lands
of the public domain; as such, they are beyond the commerce of man, as provided under
Section 2, Article XII of the Constitution.

MAIN POINT:

The 592.15 hectares of submerged areas of manila bay remain inalienable natural
resources of the public domain. Since the amended JVA seeks to transfer to AMARI, a
private corporation, ownership of 77.34 hectares of the freedom islands, such transfer is
void for being contrary to section 3, article xii of the 1987 constitution which prohibits
private corporations from acquiring any kind of alienable land of the public

CASE NO. 1264 - LAKBAO


ARTICLE XII: SECTION 3 - LANDS OF THE PUBLIC DOMAIN
REPUBLIC V. SOUTHSIDE

FACTS:
President Carlos P. Garcia issued Proclamation No. 423[5] establishing a military
reservation known as Fort William McKinley - later renamed Fort Andres Bonifacio Military
Reservation (FBMR). The proclamation "withdr[ew] from sale or settlement and reserve[d]
for military purposes, under the administration of the Chief of Staff of the [AFP] " the
[certain] parcels of the public domain [indicated in plan Psu-2031]" situated in the several
towns and a city of what was once the Province of Rizal. At the core of the instant
proceedings for declaration of nullity of title are parcels of land with a total area of 39.99
hectares, more or less, known as or are situated in what is referred to as the JUSMAG
housing area in Fort Bonifacio. As may be gathered from the pleadings, military officers,
both in the active and retired services, and their respective families, have been occupying
housing units and facilities originally constructed by the AFP on the JUSMAG area.
Private respondent SHAI is a non-stock corporation organized mostly by wives of AFP
military officers. Records show that SHAI was able to secure from the Registry of Deeds
of the Province of Rizal a title - Transfer Certificate of Title (TCT) No. 15084[12] - in its
name to the bulk of, if not the entire, JUSMAG area.

ISSUE: Whether the JUSMAG area, during the period material, alienable or inalienable,
as the case may be, and,therefore, can or cannot be subject of a lawful private
conveyance.

RULING: Private respondent SHAI parlays the "what-can-be-raised" line. It urges the
dismissal of the petition on the ground that the issues raised therein, particularly those
bearing on the authenticity of Exhibit "A"/"1," are mainly questions of fact, adding that the
matter of the inalienability of the area purportedly sold is outside the issue agreed upon
during the pre-trial stage. Petitioner Republic, as do the intervenors, asserts the
inalienable character of the JUSMAG area, the same having not effectively been
separated from the military reservation and declared as alienable and disposable.The
Republic's and the intervenor's parallel assertions are correct. The President, upon the
recommendation of the Secretary of Environment and Natural Resources, may designate
by proclamation any tract or tracts of land of the public domain as reservations for the use
of the Republic or any of its branches, or for quasi-public uses or purposes. Such tract or
tracts of land thus reserved shall be non-alienable and shall not be subject to sale or other
disposition until again declared alienable.

MP: Consistent with the foregoing postulates, jurisprudence teaches that a military
reservation, like the FBMR, or a part thereof is not open to private appropriation or
disposition and, therefore, not registrable, unless it is in the meantime reclassified and
declared as disposable and alienable public land. And until a given parcel of land is
released from its classification as part of the military reservation zone and reclassified by
law or by presidential proclamation as disposable and alienable, its status as part of a
military reservation remains, even if incidentally it is devoted for a purpose other than as
a military camp or for defense. So it must be here.

CASE NO. 1265 – SABTALUH


ARTICLE XII: SEC. – LANDS OF PUBLIC DOMAIN
REPUBLIC V.S. T.A.N. PROPERTIES, INC., {GR No. 154953, 2008-06-26}

FACTS: This case originated from an Application for Original Registration of Title filed by
T.A.N. Properties, Inc. covering Lot 10705-B of the subdivision plan Csd-04-019741
which is a portion of the consolidated Lot 10705, Cad-424, Sto. Tomas Cadastre All
adjoining owners and all government agencies and offices concerned were notified of the
initial hearing. On 15 November 1999, the trial court issued an Order of General Default
against the whole world except as against petitioner. In its 16 December 1999 Decision,
the trial court adjudicated the land in favor of respondent. Petitioner appealed from the
trial court's Decision. Petitioner alleged that the trial court erred in granting the application
for registration absent clear evidence that the applicant and its predecessors-in-interest
have complied with the period of possession and occupation as... required by law. In its
21 August 2002 Decision, the Court of Appeals affirmed in toto the trial court's Decision.

ISSUES: Whether the land is alienable and disposable.

RULING: No, Respondent Failed to Prove that the Land is Alienable and Disposable. In
this case, respondent submitted two certifications issued by the Department of
Environment and Natural Resources (DENR). The certifications are not sufficient. The
1987 Constitution absolutely prohibits private corporations from acquiring any kind of
alienable land of the public domain. The well-entrenched rule is that all lands not
appearing to be clearly of private dominion presumably belong to the State, by
incontrovertible evidence, the presumption that the land subject of an application for
registration is alienable and disposable rests with the applicant. The applicant for land
registration must prove that the DENR Secretary had approved the land classification and
released the land of the public domain as alienable and disposable, and that the land
subject of the application for registration falls within the approved area per verification
through survey by the PENRO or CENRO. In addition, the applicant for land registration
must present a copy of the original classification approved by the DENR Secretary and
certified as a true copy by the legal custodian of the official records.

MAINPOINT: All lands not appearing to be clearly of private dominion presumably belong
to the State, by incontrovertible evidence, the presumption that the land subject of an
application for registration is alienable and disposable rests with the applicant.

CASE NO: 1266 - VARGAS


ARTICLE XII: SECTION 5. ANCESTRAL LANDS AND DOMAIN
CRUZ V. SEC. OF DENR

FACTS: Petitioners Isagani Cruz and Cesar Europa filed a suit for prohibition and
mandamus as citizens and taxpayers, assailing the constitutionality of certain provisions
of Republic Act No. 8371, otherwise known as the Indigenous People’s Rights Act of 1997
(IPRA) and its implementing rules and regulations (IRR). The petitioners assail certain
provisions of the IPRA and its IRR on the ground that these amount to an unlawful
deprivation of the State’s ownership over lands of the public domain as well as minerals
and other natural resources therein, in violation of the regalian doctrine embodied in
section 2, Article XII of the Constitution.

ISSUE: Whether the provisions of IPRA contravene the Constitution?

RULING: No, the provisions of IPRA do not contravene the Constitution. Examining the
IPRA, there is nothing in the law that grants to the ICCs/IPs ownership over the natural
resources within their ancestral domain. Ownership over the natural resources in the
ancestral domains remains with the State and the rights granted by the IPRA to the
ICCs/IPs over the natural resources in their ancestral domains merely gives them, as
owners and occupants of the land on which the resources are found, the right to the small
scale utilization of these resources, and at the same time, a priority in their large scale
development and exploitation.

MAIN POINT: Ancestral lands and ancestral domains are not part of the lands of the
public domain. They are private lands and belong to the ICCs/IPs by native title, which is
a concept of private land title that existed irrespective of any royal grant from the State.
However, the right of ownership and possession by the ICCs/IPs of their ancestral
domains is a limited form of ownership and does not include the right to alienate the same.
CASE NO. 1267 - TIPOE
ARTICLE XII:SEC 6 - COMMON GOOD
TELECOM & GMA NETWORKS, INC. V. COMELEC [G.R. NO. 132922. APRIL 21,
1998.]

FACTS: Section 11 (b) of R.A. No. 6646 (Ban on Paid Political Ads) prohibits the sale or
donation of print space or air time for political ads, except to the Commission on Elections.

ISSUE: W/N such provision constitutes taking of property without due process of law and
without just compensation.

RULING: NO. Radio and television broadcasting companies, which are given franchises,
do not own the airwaves and frequencies through which they transmit broadcast signals
and images. They are merely given the temporary privilege of using them.

MAIN POINT: Since a franchise is a mere privilege, the exercise of the privilege may
reasonably be burdened with the performance by the grantee of some form of public
service. Under Sec. 6 of Article XII of the Constitution, such privilege use of property is
subject to the duty of the State to promote distributive justice and to intervene when the
common good so demands. It is for the common good to grant free air time for the
candidates but even more of the public, particularly the voters, so that they will be fully
informed of the issues in an election.

CASE NO. 1268 - CHIONG


ARTICLE XII: SECTION 7 - PRIVATE LANDS
REPUBLIC VS. CA
FACTS:
on June 17, 1978, respondent spouses bought lots if 347 and 348, Cad. 91.77 sq situated
in San Pablo City, from one Cristeta Dazo Belen whom inherited the land from her father,
Pedro Dazo who has been in possession of the lot since 1937- corroborated by the sister
of Cristeta that taxes were paid and their father had introduced improvements and that
Cristeta inherited it. There was also a report from the Bureau of Lands together with a
letter from the Bureau of Forest Development, to prove that the questioned lots were part
of the alienable and disposable zone of the government.
At the time of the purchase, respondent spouses were then natural-born Filipino Citizens.
On February 5 1987, the spouses filed an application for the registration of the title of the
two(2) parcels of land before the Regional Trial Court of San Pablo City. This time,
however, they were no longer Filipino Citizens. They’re already Canadian Citizens thru
naturalization. RTC granted the application. CA also affirmed the decision.
ISSUE:
Whether or not, the applicant considering that he is a foreign national can apply for the
registration of title over a parcel of lot he acquired when he was still a Filipino citizen.
RULING:
Yes. SC ruled that, it is undisputed that private respondents were undoubtedly natural-
born Filipino citizens at the time of the acquisition of the properties and by virtue thereof,
acquired vested rights thereon, tacking in the process, the possession in the concept of
and the prescribed period of time held by their predecessors-in-interest under the Public
Land Act. Their purpose in initiating the instant action is merely to confirm their title over
the land, for, as has been passed upon, they had been the owners of the same since
1978.
MAIN POINT:
A foreign national may apply for registration of title over a parcel of land which he acquired
by purchase while still a citizen of the philippines, from a vendor who has complied with
the requirements for registration under the public land act. The time to determine whether
the person acquiring land is qualified is the time the right to own land is acquired and not
the time to register ownership.

CASE NO. 1269 - LAKBAO


Article XII. National Economy and Patrimony. Section 7. Private Lands
Zaragosa v. CA
FACTS:
On December 9, 1964, Flavio Zaragoza Cano died without a will and was survived by his
four children: Gloria, Zacariaz, Florentino and Alberta, all surnamed Zaragoza. On
December 28, 1981, private respondent Alberta Zaragoza-Morgan filed a complaint with
CFI against Spouses Florentino and Erlinda, herein petitioners, for delivery of her
inheritance share, consisting of Lots 943 and 871, and for payment of damages. She
alleged that her father, in his lifetime, partitioned the aforecited properties among his four
children. The shares of her brothers and sister were given to them in advance by way of
deed of sale, but without valid consideration, while her share, which consists of lots no.
871 and 943, was not conveyed by way of deed of sale then. She averred that because
of her marriage, she became an American citizen and was prohibited to acquire lands in
the Philippines except by hereditary succession. For this reason, no formal deed of
conveyance was executed in her favor covering these lots during her father’s lifetime

ISSUE: Whether or not the partition inter vivos by Flavio Zaragoza Cano of his
properties, which include Lots 871 and 943, is valid.

RULING: YES. The Court held that it is basic in the law of succession that a partition inter
vivos may be done for as long as legitimes are not prejudiced. Art. 1080 of the Civil Code
is clear on this. Thus, the court ruled that that during the lifetime of Flavio, he already
partitioned and distributed his properties among his three children, excepting private
respondent, through deeds of sale. A deed of sale was not executed in favor of private
respondent because she had become an American citizen and the Constitution prohibited
a sale in her favor. Petitioner admitted Lots 871 and 943 were inheritance shares of the
private respondent. As to the alleged prejudice on legitimes, the Court stated that the
legitime of compulsory heirs is determined after collation, as provided for in Article 1061:
Every compulsory heir, who succeeds with other compulsory heirs, must bring into the
mass of the estate any property or right which he may have received from the decedent,
during the lifetime of the latter, by way of donation, or any other gratuitous title in order
that it may be computed in the determination of the legitime of each heir, and in the
account of the partition. Unfortunately, collation cannot be done in this case where the
original petition for delivery of inheritance share only impleaded one of the other
compulsory heirs. The petition must therefore be dismissed without prejudice to the
institution of a new proceeding where all the indispensable parties are present for the
rightful determination of their respective legitime and if the legitimes were prejudiced by
the partitioning inter vivos.
MP:

CASE NO. 1270 – SABTALUH


ARTICLE XII: SEC. 7 – PRIVATE LANDS
Ramirez v. Ramirez {GR No. L-27962, February 15, 1982}

FACTS: Jose Eugenio Ramirez, a Filipino national, died in Spain with only his French
widow as compulsory heir. His will was admitted to probate by the Court of First Instance
of Manila, Branch X. The administrator of the estate submitted a project of partition giving
one part of the estate to the widow “en pleno dominio” in satisfaction of her legitime while
the other part of the “free portion” to his two grandnephews Roberto and Jorge Ramirez.
Furthermore, one third of the free portion is charged with the widow’s usufruct and the
remaining two thirds (2/3) with a usufruct in favor of Wanda Wrobleski and Austrian
national. Jorge and Roberto Ramirez opposed the project of partition, as well as the
substitutions provided by the testator as to the usufructs of the widow and of Wanda on
the basis that it violates the sec 7, of article 12 of the constitution.
ISSUE: Whether or not the will on the usufructuary rights over a real property to an alien
violates the constitution.
RULING:No. the Court upholds the usufruct in favour of Wanda because a usufruct does
not vest title to the land in the usufructuary and it is the vesting of title to aliens which is
proscribed by the Constitution.
MAINPOINT: Usufruct albeit a real right does not vest title to the land in usufructuary and
it is the vesting of the title to the land in favor of the aliens which is proscribed by the
constitution
CASE NO: 1271 - VARGAS
ARTICLE XII: SECTION 7- PRIVATE LANDS
HALILI V. CA

FACTS: Simeon de Guzman, an American citizen, died sometime in 1968, leaving real
properties in the Philippines. His forced heirs were his widow private respondent Helen
Meyers Guzman, and his son, private respondent David Rey Guzman, both of whom are
also American citizens. On August 9, 1989, Helen executed a deed of quitclaim,
assigning, transferring and conveying to David Rey all her rights, titles and interests in
and over six parcels of land which the two of them inherited from Simeon. Among the said
parcels of land is that now in litigation Guzman then sold the parcel of land to Catanaig,
who is one of respondents in this case. Petitioners, who are owners of the adjoining lot,
filed a complaint before the Regional Trial Court of Malolos, Bulacan, questioning the
constitutionality and validity of the two conveyances — between Helen Guzman and
David Rey Guzman, and between the latter and Emiliano Cataniag — and claiming
ownership thereto based on their right of legal redemption
under Art. 1621 of the Civil Code. The trial court dismissed the complaint. The CA denied
the appeal of the Halilis.

ISSUE: Whether or not the sale of the land is null and void

RULING: No, because the prohibition in the constitution has already been served. Article
XII, Section 7 provides that Non- Filipinos cannot acquire or hold title to private lands or
to lands of the public domain, in fine, non-Filipinos cannot acquire or hold title to private
lands or to lands of the public domain, except only by way of legal succession. While it is
true that the transfer of Helen of his right to her son who is an American citizen contradicts
the prohibition set forth in the Constitution, the Supreme Court upheld the subsequent
sale of the land to Catanig, a Filipino citizen.

MAIN POINT: Jurisprudence is consistent that “if land is invalidly transferred to an alien
who subsequently becomes a citizen or transfers it to a citizen, the flaw in the original
transaction is considered cured and the title of the transferee is rendered valid.”

CASE NO. 1272 - TIPOE


ART. XII SEC. 7: PRIVATE LANDS
LEE V. REPUBLIC [G.R. NO. 128195. OCTOBER 3, 2001.]

FACTS: Petitioners, Filipino heirs of Lee Liong, sought reconstitution of title of subject
parcel of land in the name of Lee Liong whose transfer certificate of title was lost or
destroyed during the war. At the trial, they presented documents proving the sale of the
land from the Dinglasans to Lee Liong. The Dinglasans, however, claimed that Lee Liong,
a Chinese citizen, was disqualified to acquire the land in question.

ISSUE: W/N Lee Liong’s citizenship disqualifies the heirs’ right to the land property.
RULING: NO. If land is invalidly transferred to an alien who subsequently becomes a
citizen or transfers it to a citizen, the flaw in the original transaction is considered cured
and the title of the transferee is rendered valid."

MAIN POINT: In sales of real estate to aliens incapable of holding title thereto by virtue
of the provisions of the Constitution both the vendor and the vendee are deemed to have
committed the constitutional violation and being thus in pari delicto the courts will not
afford protection to either party. The constitutional proscription on alien ownership of
lands of the public or private domain was intended to protect lands from falling in the
hands of non-Filipinos. In this case, however, there would be no more public policy
violated since the land is in the hands of Filipinos qualified to acquire and own such land.

CASE NO. 1273 - CHIONG


ARTICLE XII: SECTION 7. PRIVATE LANDS
FRENZEL V. CATITO
FACTS:
Petitioner Alfred Fritz Frenzel is an Australian citizen of German descent. He was so
enamored with Ederlina that he bought her numerous properties such as house and lot
in Quezon City and in Davao City. He also put up a beauty parlor business in the name
of Ederlina. Alfred was unaware that Ederlina was married until her spouse Klaus Muller
wrote a letter to Alfred begging the latter to leave her wife alone.
When Alfred and Ederlinas relationship started deteriorating. Ederlina had not been able
to secure a divorce from Klaus. The latter could charge her for bigamy and could even
involve Alfred, who himself was still married. To avoid complications, Alfred decided to
live separately from Ederlina and cut off all contacts with her. On October 15, 1985, Alfred
wrote to Ederlinas father, complaining that Ederlina had taken all his life savings and
because of this, he was virtually penniless. He further accused the Catito family of
acquiring for themselves the properties he had purchased with his own money. He
demanded the return of all the amounts that Ederlina and her family had stolen and turn
over all the properties acquired by him and Ederlina during their coverture.
ISSUE:
Whether the petitioner could recover the money used in purchasing the several
properties
RULING:
No, even if, as claimed by the petitioner, the sales in question were entered into by him
as the real vendee, the said transactions are in violation of the Constitution; hence, are
null and void ab initio. A contract that violates the Constitution and the law, is null and
void and vests no rights and creates no obligations. It produces no legal effect at all. The
petitioner, being a party to an illegal contract, cannot come into a court of law and ask to
have his illegal objective carried out. One who loses his money or property by knowingly
engaging in a contract or transaction which involves his own moral turpitude may not
maintain an action for his losses. To him who moves in deliberation and premeditation,
the law is unyielding. The law will not aid either party to an illegal contract or agreement;
it leaves the parties where it finds them.
MAIN POINT:
A contract that violates the constitution and the law, is null and void and vests no rights
and creates no obligations. It produces no legal effect at all. Alfred, being a party to an
illegal contract, cannot come into a court of law and ask to have his illegal objective carried
out. One who loses his money or property by knowingly engaging in a contract or
transaction which involves his own moral turpitude may not maintain an action for his
losses. To him who moves in deliberation and premeditation, the law is unyielding. The
law will not aid either party to an illegal contract or agreement; it leaves the parties where
it finds them

CASE NO. 1274 - LAKBAO


Article XII. National Economy and Patrimony. Section 7. Private Lands
Lentfer v. Wolff

Facts:
Issue:
Ruling:
MP:

CASE NO. 1275 – SABTALUH


ARTICLE XII: SEC. 7 – PRIVATE LANDS
MULLER VS MULLER
FACTS: Petitioner Elena Buenaventura Muller and respondent Helmut Muller were
married in Hamburg, Germany on September 22, 1989. The couple resided in Germany
at a house owned by respondent’s parents but decided to move and reside permanently
in the Philippines. respondent had inherited the house in Germany for which he sold and
used the proceeds for the purchase of a parcel of land in Antipolo, Rizal and the
construction of a house. The Antipolo property was registered in the name of petitioner,
Elena Buenaventura Muller. The spouses eventually separated. On September 26, 1994,
respondent filed a petition for separation of properties before the Regional Trial Court of
Quezon City. The court granted said petition. With regard to the Antipolo property, the
court held that it was acquired using paraphernal funds of the respondent. However, it
ruled that respondent cannot recover his funds because the property was purchased in
violation of Section 7, Article XII of the Constitution. The court of appeals reversed the
decision of the RTC. It ordered the respondent to REIMBURSE the petitioner. Elena
Muller then filed a petition for review on certiorari.
ISSUE: Whether or not respondent Helmut Muller is entitled to reimbursement.
RULING: No, respondent Helmut Muller is not entitled to reimbursement. There is an
express prohibition against foreigners owning land in the Philippines. Art. XII, Sec. 7 of
the 1987 Constitution provides: “Save in cases of hereditary succession, no private lands
shall be transferred or conveyed except to individuals, corporations, or associations
qualified to acquire or hold lands of the public domain.” In the case at bar, the respondent
willingly and knowingly bought the property despite a constitutional prohibition. And to get
away with that constitutional prohibition, he put the property under the name of his Filipina
wife.
MAINPOINT: Transactions that are in violation of the constitution; are null and void ab
initio. A contract that violates the constitution is null and void and vests no right or
obligations.
CASE NO: 1276 - VARGAS
ARTICLE XII: SECTION 7 - PRIVATE LANDS
MULLER V. MULLER

FACTS: Petitioner Elena Buenaventura Muller and respondent Helmut Muller were
married in Hamburg, Germany on September 22, 1989. The couple resided in Germany
at a house owned by respondent’s parents but decided to move and reside permanently
in the Philippines in 1992. By this time, respondent had inherited the house in Germany
from his parents which he sold and used the proceeds for the purchase of a parcel of land
in Antipolo, Rizal at the cost of P528,000.00 and the construction of a house amounting
to P2,300,000.00. The Antipolo property was registered in the name of petitioner, Elena
Buenaventura Muller.
Due to incompatibilities and respondents alleged womanizing, drinking, and
maltreatment, the spouses eventually separated.
On September 26, 1994, respondent filed a petition for separation of properties before
the Regional Trial Court of Quezon City. The court granted said petition. It also decreed
the separation of properties between them and ordered the equal partition of personal
properties located within the country, excluding those acquired by gratuitous title during
the marriage. With regard to the Antipolo property, the court held that it was acquired
using paraphernal funds of the respondent. However, it ruled that respondent cannot
recover his funds because the property was purchased in violation of Section 7, Article
XII of the Constitution.
The respondent elevated the case to the Court of Appeals, which reversed the decision
of the RTC. It held that respondent merely prayed for reimbursement for the purchase of
the Antipolo property, and not acquisition or transfer of ownership to him. It ordered the
respondent to REIMBURSE the petitioner the amount of P528,000.00 for the acquisition
of the land and the amount of P2,300,000.00 for the construction of the house situated in
Antipolo, Rizal.
Elena Muller then filed a petition for review on certiorari.

ISSUE: Whether or not respondent Helmut Muller is entitled to reimbursement.


RULING: No, respondent Helmut Muller is not entitled to reimbursement. the respondent
willingly and knowingly bought the property despite a constitutional prohibition. And to get
away with that constitutional prohibition, he put the property under the name of his Filipina
wife. He tried to do indirectly what the fundamental law bars him to do directly.
With this, the Supreme Court ruled that respondent cannot seek reimbursement on the
ground of equity. It has been held that equity as a rule will follow the law and will not
permit that to be done indirectly which, because of public policy, cannot be done directly.

MAIN POINT: There is an express prohibition against foreigners owning land in the
Philippines. Art. XII, Sec. 7 of the 1987 Constitution provides: “Save in cases of hereditary
succession, no private lands shall be transferred or conveyed except to individuals,
corporations, or associations qualified to acquire or hold lands of the public domain.”

CASE NO. 1277 - TIPOE


ART. XII SEC. 7: PRIVATE LANDS
MATTHEWS V. TAYLOR SPOUSES [G.R. NO. 164584. JUNE 22, 2009.]

FACTS: Joselyn as lessor and petitioner Philip Matthews as lessee, entered into an
Agreement of Lease involving a Boracay property for a period of 25 years. Petitioner
thereafter took possession of the property and renamed the resort as Music Garden
Resort. Benjamin, the alien husband of Joselyn, claimed that the Agreement was null and
void since it was entered into by Joselyn without his consent alleging that he should not
be deprived of the prerogative of a husband in respect of conjugal property.

ISSUE: W/N respondent, as the husband of the lessor Joselyn, has legal standing to
declare such agreement of lease as invalid.

RULING: NO. Considering that Joselyn appeared to be the designated "vendee" in the
Deed of Sale of said property, she acquired sole ownership thereto. This is true even if
we sustain Benjamin's claim that he provided the funds for such acquisition. To sustain
such a theory would countenance indirect contravention of the constitutional prohibition.
If the property were to be declared conjugal, this would accord the alien husband a
substantial interest and right over the land, as he would then have a decisive vote as to
its transfer or disposition. This is a right that the Constitution does not permit him to have.

MAIN POINT: An alien, since he has no right to acquire private land, has no right to
challenge the validity of the lease of a piece of land which his wife had acquired.
CASE NO. 1278 - CHIONG
ARTICLE XII: SECTION 7 - PRIVATE LANDS
HULST VS. PR BUILDERS
FACTS:
Jacobus Bernhard and his wife, both Dutch nationals entered into a contract to sell with
PR Builders for the purchase of 210 sq m residential unit in Laurel, Batangas. PR Builders
failed to comply with their verbal promise to complete the project by June, and the
petitioner filed before the Housing and Land Use Regulatory Board (HLURB), a complaint
for rescission of contract with interest, damages, and attorney's fees. The HLURB Arbiter
issued a Writ of Execution addressed to Ex-Officio Sheriff of Batangas to execute his
judgment. They require the Sheriff to levy first on respondents personal property but
unsatisfied so the Sheriff levied on respondent's 15 parcel of land. The respondent filed
an urgent motion to Quash the Writ of levy on the ground that the Sheriff made an overlevy
since the aggregate value of the property at 6500/sqm is P83,616,000 which was over
and above the judgment award. The Sheriff continue the auction and the 15 parcel of land
was sold to Holly Properties Realty Corporation for the amount of 5,450,653. The sum
5,313,040 was turned to petitioner in satisfaction of his judgment award after deducting
all the legal fees. The HLURB Arbiter and Director authorized the Sheriff to set aside the
levy of the said property because of its inadequacy of the price.
ISSUES:
Whether or not the contract to sell between Hulst and PR Builders is valid.
RULING:
No. The contract to sell between Hulst and PR Builders is NULL and VOID. According to
Sec.7 Art. XII of 1987 Constitution, "no private lands shall be transferred or conveyed
except to individuals, corporations, or associations qualified to acquire or hold lands of
public domain". Since the petitioner and his wife are Dutch nationals, they are disqualified
to acquire private lands. The petitioner is entitled only to recover what he has paid so he
must return to respondent the amount P2,125,540 without interest in excess to the
proceeds of the auction sale. A void contract is equivalent to nothing; it produces no civil
effect.
MAIN POINT:
No condominium unit therein shall be conveyed or transferred to persons other than
Filipino citizens or corporations at least 60% of the capital stock of which belong to Filipino
citizens, except in cases of hereditary succession. Where the common areas in a
condominium project are held by a corporation, no transfer or conveyance of a unit shall
be valid if the concomitant transfer of the appurtenant membership or stockholding in the
corporation will cause the alien interest in such corporation to exceed the limits imposed
by existing laws.
CASE NO. 1279 - LAKBAO
ARTICLE XII: SECTION 7- PRIVATE LANDS
TING HO V. TENG

FACTS: Felix Ting Ho, Jr., Merla Ting Ho Braden, Juana Ting Ho and Lydia Ting Ho
Belenzo against their brother, respondent Vicente Teng Gui. The controversy revolves
around a parcel of land, and the improvements which should form part of the estate of
their deceased father, Felix Ting Ho, and should be partitioned equally among each of
the siblings. Petitioners alleged that their father Felix Ting Ho died intestate on June 26,
1970, and left upon his death an estate. According to petitioners, the said lot and
properties were titled and tax declared under trust in the name of respondent Vicente
Teng Gui for the benefit of the deceased Felix Ting Ho who, being a Chinese citizen, was
then disqualified to own public lands in thePhilippines; and that upon the death of Felix
Ting Ho, the respondent took possession of the same for his own exclusive use and
benefit to their exclusion and prejudice.

ISSUE: Whether or not the sale was void.

RULING: No, the sale was not void. Article 1471 of the Civil Code has provided that if the
price is simulated, the sale is void, but the act may be shown to have been in reality a
donatin, or some other act or contract. The sale in this case, was however valid because
the sale was in fact a donation. The law requires positive proof of the simulation of the
price of the sale. But since the finding was based on a mere assumption, the price has
not been proven to be a simulation.
MP:

CASE NO. 1280- SABTALUH


ARTICLE XII: PRIVATE LANDS
HULST v. PR BUILDERS, {GR No. 156364, 2008-09-25}

FACTS: Jacobus Bernhard Hulst (petitioner) Dutch national, filed the present Motion for
Partial Reconsideration after he was ordered to return to respondent the amount of
P2,125,540.00 in excess of the proceeds of the auction sale delivered to petitioner.
Petitioner contends that the Contract to sell between petitioner and respondent involved
a condominium unit and did not violate the Constitutional proscription against ownership
of land by aliens. He argues that the contract to sell will not transfer to the buyer ownership
of the land on which the unit is situated; thus, the buyer will not get a transfer certificate
of title but merely a Condominium Certificate of Title as evidence of ownership; a perusal
of the contract will show that what the buyer acquires is the seller's title and rights to and
interests in the unit and the common areas.

ISSUE: Whether or not a foreigner can own a condominium unit.


RULING: Under Republic Act (R.A.) No. 4726, otherwise known as the Condominium
Act, foreign nationals can own Philippine real estate through the purchase of
condominium units or townhouses constituted under the Condominium principle with
Condominium Certificates of Title. Where the common areas in a condominium project
are held by a corporation, no transfer or conveyance of a unit shall be valid if the
concomitant transfer of the appurtenant... membership or stockholding in the corporation
will cause the alien interest in such corporation to exceed the limits imposed by existing
laws.
The law provides that no condominium unit can be sold without at the same time selling
the corresponding amount of rights, shares or other interests in the condominium
management body, the Condominium Corporation; and no one can buy shares in a
Condominium Corporation without at... the same time buying a condominium unit.
It expressly allows foreigners to acquire condominium units and shares in condominium
corporations up to not more than 40% of the total and outstanding capital stock of a
Filipino-owned or controlled corporation. Under this set up, the... ownership of the land is
legally separated from the unit itself. The land is owned by a Condominium Corporation
and the unit owner is simply a member in this Condominium Corporation.

MAINPOINT: A foreigner may own a condominium unit because the prohibition on aliens
is only from acquiring land. The land on which the condominium stands is own by the
condominium corporations.
CASE NO: 1281 - VARGAS
ARTICLE XII:SECTION 7. PRIVATE LANDS
OSMENA V. OSMENA

FACTS: Petitioner asserts that she is a co-owner of the three litigated properties. She
argues that the two lots were her mother’s properties and were part of the inheritance that
she and her siblings received upon Chiong Tan Sy’s death. She claims that the lots were
placed in the name of her brother Ignacio merely because their mother, a Chinese
national, was prohibited by law to own land in the Philippines.
With regard to the house, it is petitioner’s position that ownership of her share in the
ancestral home was transferred to her brother under the guise of a simulated contract to
defeat any claims by her estranged husband. As proof of her co-ownership of the house,
petitioner maintains that she has never been charged rent by her brother for her continued
residence in the same.
Respondents, on the other hand, predicate their claim to the disputed properties on the
transfer certificates of title covering the lots issued in their father’s name and a deed of
sale dated April 26, 1982 signed by petitioner herself, covering her share in the ancestral
house. Both the trial court and the Court of Appeals (CA) recognized the validity of said
documents and rendered judgment in favor of respondents. The trial court enjoined
petitioner from utilizing the litigated land for her orchid business and ordered her to leave
the house immediately. The CA modified the decision by declaring petitioner a co-owner
of the litigated ancestral house to the extent of the shares she inherited from two of her
siblings.
ISSUE: Whether the CA erred in giving credence to the deed of sale dated April 26, 1982
and in holding that respondents are the owners of the disputed lots

RULING: Thoroughly reviewed the records of this case and agree that the deed of sale
dated April 26, 1982 is a legal and binding document. The testimonies of the witnesses
to the document attest to the parties freely signing the document and the occurrence of
the transaction in a clear and definite manner. Moreover, it is a notarized document which
renders it a prima facie evidence of the facts contained therein. In the absence of
documents or testimonies from disinterested persons proving petitioner’s claim of a
fictitious sale, there is no basis to set aside the deed of sale.
In petitions for review on certiorari, the jurisdiction of this Court is limited to the review
and revision of errors of law allegedly committed by the appellate court inasmuch as the
latter’s findings of fact are deemed conclusive. Given that the facts of this case, as
gleaned from the records, fully support the decision of the trial court and the CA, we see
no valid reason to overturn the findings of the courts below and therefore sustain the
judgment of the appellate court.

MAIN POINT: This Court has oft repeated that he who comes to court must come with
clean hands. Considering that the right over the litigated properties claimed by petitioner
stems allegedly from illegal acts, no affirmative relief of any kind is available.

CASE NO. 1282 - TIPOE


ART. XII SEC. 7: PRIVATE LANDS
BEUMER V. AMORES [G.R. NO. 195670. DECEMBER 3, 2012.]

FACTS: The marriage between petitioner Willem Buemer, a Dutch, and respondent
Avelina Amores, a Filipina was declared null on the basis of the former’s psychological
incapacity. Consequently, in the Petition for Dissolution of Conjugal Partnership, included
were lots registered in the name of the respondent which she maintained were purchased
using exclusive personal funds but the petitioner claims that these properties were
acquired with the money he received from the Dutch government as his disability benefit
since respondent did not have sufficient income to pay for their acquisition.

ISSUE: W/N the petitioner can claim rightful ownership of said lots.

RULING: NO. It is clear that he willingly and knowingly bought the property despite the
prohibition against foreign ownership of Philippine land enshrined under Section 7, Article
XII of the 1987 Philippine Constitution.

MAIN POINT: Petitioner openly admitted that he is well aware of the constitutional
prohibition on foreign ownership of private lands and even affirmed that, because of such
prohibition, he and respondent registered the subject properties in the latter's name.
Petitioner's actuations showed his palpable intent to skirt the constitutional prohibition.
CASE NO. 1283 - CHIONG
ARTICLE XII: SECTION 8. EXCEPTION FOR FORMER FILIPINO CITIZENS
REPUBLIC VS. CA
FACTS:
on June 17, 1978, respondent spouses bought lots if 347 and 348, Cad. 91.77 sq situated
in San Pablo City, from one Cristeta Dazo Belen whom inherited the land from her father,
Pedro Dazo who has been in possession of the lot since 1937- corroborated by the sister
of Cristeta that taxes were paid and their father had introduced improvements and that
Cristeta inherited it. There was also a report from the Bureau of Lands together with a
letter from the Bureau of Forest Development, to prove that the questioned lots were part
of the alienable and disposable zone of the government.
At the time of the purchase, respondent spouses were then natural-born Filipino Citizens.
On February 5 1987, the spouses filed an application for the registration of the title of the
two(2) parcels of land before the Regional Trial Court of San Pablo City. This time,
however, they were no longer Filipino Citizens. They’re already Canadian Citizens thru
naturalization. RTC granted the application. CA also affirmed the decision.
ISSUE:
Whether or not, the applicant considering that he is a foreign national can apply for the
registration of title over a parcel of lot he acquired when he was still a Filipino citizen.
RULING:
Yes. SC ruled that, it is undisputed that private respondents were undoubtedly natural-
born Filipino citizens at the time of the acquisition of the properties and by virtue thereof,
acquired vested rights thereon, tacking in the process, the possession in the concept of
and the prescribed period of time held by their predecessors-in-interest under the Public
Land Act. Their purpose in initiating the instant action is merely to confirm their title over
the land, for, as has been passed upon, they had been the owners of the same since
1978.

CASE NO. 1284 - LAKBAO


ARTICLE XII: SECTION 10. FILIPINIZATION
MANILA PRINCE HOTEL V. GSIS

FACTS: The Government Service Insurance System (GSIS) decided to sell through
public bidding 30% to 51% of the issued and outstanding shares of the Manila Hotel
(MHC). In a close bidding, two bidders participated: Manila Prince Hotel Corporation
(MPHC), a Filipino corporation, which offered to buy 51% of the MHC at P41.58 per share,
and Renong Berhad, a Malaysian firm, with ITT-Sheraton as its hotel operator, which bid
for the same number of shares at P44.00 per share, or P2.42 more than the bid of
petitioner. Pending the declaration of Renong Berhard as the winning bidder and the
execution of the contracts, the MPHC matched the bid price in a letter to GSIS. MPHC
sent a manager’s check to the GSIS in a subsequent letter, which GSIS refused to accept.
On 17 October 1995, perhaps apprehensive that GSIS has disregarded the tender of the
matching bid, MPHC came to the Court on prohibition and mandamus. Petitioner invokes
Sec. 10, second par., Art. XII, of the 1987 Constitution and submits that the Manila Hotel
has been identified with the Filipino nation and has practically become a historical
monument which reflects the vibrancy of Philippine heritage and culture. Respondents
assert that Sec. 10, second par., Art. XII, of the 1987 Constitution is merely a statement
of principle and policy since it is not a self-executing provision and requires implementing
legislation(s).

ISSUE: Whether the provisions of the Constitution, particularly Article XII Section 10, are
self-executing and the Manila Hotel forms part of the national patrimony.

RULING: Yes. Sec 10, Art. XII of the 1987 Constitution is a self-executing provision.
Hence, unless it is expressly provided that a legislative act is necessary to enforce a
constitutional mandate, the presumption now is that all provisions of the constitution are
self-executing. If the constitutional provisions are treated as requiring legislation instead
of self-executing, the legislature would have the power to ignore and practically nullify the
mandate of the fundamental law. In fine, Section 10, second paragraph, Art. XII of the
1987 Constitution is a mandatory, positive command which is complete in itself and which
needs no further guidelines or implementing laws or rules for its enforcement. From its
very words the provision does not require any legislation to put it in operation. In its plain
and ordinary meaning, the term patrimony pertains to heritage. When the Constitution
speaks of national patrimony, it refers not only to the natural resources of the Philippines,
as the Constitution could have very well used the term natural resources, but also to the
cultural heritage of the Filipinos.

MP: A provision which lays down a general principle, such as those found in Article II of
the 1987 Constitution, is usually not self-executing. But a provision which is complete in
itself and becomes operative without the aid of supplementary or enabling legislation, or
that which supplies sufficient rule by means of which the right it grants may be enjoyed
or protected, is self-executing.

Case No. 1285 - SABTALUH

CASE NO: 1286 - VARGAS


ARTICLE XII: SECTION 10. FILIPINIZATION
TANADA V. ANGARA

FACTS: This is a case petition by Sen. Wigberto Tanada, together with other lawmakers,
taxpayers, and various NGO’s to nullify the Philippine ratification of the World Trade
Organization (WTO) Agreement.
Petitioners believe that this will be detrimental to the growth of our National Economy and
against to the “Filipino First” policy. The WTO opens access to foreign markets, especially
its major trading partners, through the reduction of tariffs on its exports, particularly
agricultural and industrial products. Thus, provides new opportunities for the service
sector cost and uncertainty associated with exporting and more investment in the country.
These are the predicted benefits as reflected in the agreement and as viewed by the
signatory Senators, a “free market” espoused by WTO.
Petitioners also contends that it is in conflict with the provisions of our constitution, since
the said Agreement is an assault on the sovereign powers of the Philippines because it
meant that Congress could not pass legislation that would be good for national interest
and general welfare if such legislation would not conform to the WTO Agreement.

ISSUE: Whether or not the provisions of the ‘Agreement Establishing the World Trade
Organization and the Agreements and Associated Legal Instruments included in Annexes
one (1), two (2) and three (3) of that agreement’ cited by petitioners directly contravene
or undermine the letter, spirit and intent of Section 19, Article II and Sections 10 and 12,
Article XII of the 1987 Constitution.

RULING: No. While the Constitution indeed mandates a bias in favor of Filipino goods,
services, labor and enterprises, at the same time, it recognizes the need for business
exchange with the rest of the world on the bases of equality and reciprocity and limits
protection of Filipino enterprises only against foreign competition and trade practices that
are unfair. In other words, the Constitution did not intend to pursue an isolationist policy.
It did not shut out foreign investments, goods and services in the development of the
Philippine economy. While the Constitution does not encourage the unlimited entry of
foreign goods, services and investments into the country, it does not prohibit them either.
In fact, it allows an exchange on the basis of equality and reciprocity, frowning only on
foreign competition that is unfair.

MAIN POINT: The alleged impairment of sovereignty in the exercise of legislative and
judicial powers is balanced by the adoption of the generally accepted principles of
international law as part of the law of the land and the adherence of the Constitution to
the policy of cooperation and amity with all nations. The Senate, after deliberation and
voting, voluntarily and overwhelmingly gave its consent to the WTO Agreement thereby
making it “a part of the law of the land” is a legitimate exercise of its sovereign duty and
power.

Case No. 1287 - TIPOE


ART. XII Sec. 10: FILIPINIZATION
JG SUMMIT HOLDINGS, INC. v. CA [G.R. No. 124293. November 20, 2000.]

FACTS: Investment and Development Corporation (NIDC) and KAWASAKI of Japan


entered into a Joint Venture Agreement (JVA) for the construction, operation, and
management of Subic National Shipyard, a public utility. At the pre-bidding conference,
interested bidders were given copies of the JVA and the Asset Specific Bidding Rules
(ASBR) drafted. Paragraph 15.0 of the ASBR provides that: "In the event that the winning
bidder is a 100% foreign-owned corporation, it may name its nominee corporation to
whom the NG shares shall be conveyed, provided it owns 40% equity in the nominee
corporation, so as not to affect PHILSECO's qualification to own real estate properties in
the Philippines."

ISSUE: W/N par. 15.0 of the ASBR is constitutional.

RULING: NO. This rule is fraught with dangerous implications. It allows a completely
foreign corporation to participate in the public bidding of more than 60% of the total shares
of a public utility corporation without setting a period within which the foreign bidder should
name its nominee.

MAIN POINT: Under the Constitution, the operation of a public utility may be granted only
to Filipino citizens or associations organized under Philippine laws at least 60% of whose
capital is owned by such citizens. The participation of foreign investors is limited to their
proportionate maximum share of 40%. Par. 15.0 of the ASBR allows a totally foreign
investor to engage in the business of operating a public utility for an unlimited period of
time in total disregard of the constitutional proscription on the percentage of Filipino
ownership of corporations engaged therein. Paragraph 15.0 of the ASBR is thus directly
and openly repugnant to the Constitution considering that it allows foreign corporations
to operate a public utility for an unlimited period of time.

CASE NO. 1288 - CHIONG


ARTICLE XII: SECTION 11. PUBLIC UTILITIES
BAGATSING VS COMMITTEE (1995)
FACTS:
PETRON was originally registered with the Securities and Exchange Commission (SEC)
in 1966 under the corporate name "Esso Philippines, Inc." (ESSO) as a subsidiary of Esso
Eastern, Inc. and Mobil Petroleum Company, Inc.
In 1973, at the height of the world-wide oil crisis brought about by the Middle East
conflicts, the Philippine government acquired ESSO through the PNOC. ESSO became
a wholly-owned company of the government under the corporate name PETRON and as
a subsidiary of PNOC. The petition asked for the issuance of a temporary restraining
order to stop respondents from selling the 40% block to a foreign buyer. Petitioners
contend that PETRON is a public utility, in which foreign ownership of its equity shall not
exceed 40% thereof and the foreign participation in the governing body shall be limited to
their proportionate share in its capital. According to petitioners, ARAMCO is entitled only
to a maximum of four seats in the ten-man board but was given five seats.
ISSUE:
Wether or not Petron is a public utility
RULING:
A "public utility" under the Constitution and the Public Service Law is one organized "for
hire or compensation" to serve the public, which is given the right to demand its service.
PETRON is not engaged in oil refining for hire and compensation to process the oil of
other parties. Therefore, it is not considered a public utility,
MAIN POINT:
A "public utility" under the Constitution and the Public Service Law is one organized "for
hire or compensation" to serve the public, which is given the right to demand its service.
PETRON is not engaged in oil refining for hire and compensation to process the oil of
other parties. Likewise, the activities considered as "public utility" under Section 7 of R.A.
No. 387 refer only to petroleum which is indigenous to the Philippines. Hence, the refining
of petroleum products sourced from abroad as is done by Petron, is not within the
contemplation of the law

CASE NO. 1289 - LAKBAO


ARTICLE XII: SECTION 11 - PUBLIC UTILITIES
ALBANO V. REYES

FACTS: The Philippine Ports Authority (PPA) board directed the PPA management to
prepare for the public bidding of the development, management and operation of the
Manila International Container Terminal (MICT) at the Port of Manila. A Bidding
Committee was formed by the DOTC for the public bidding. After evaluation of several
bids, the Bidding Committee recommended the award of the contract to respondent
International Container Terminal Services, Inc. (ICTSI). Accordingly, Rainerio Reyes,
then DOTC secretary, declared the ICTSI consortium as the winning bidder. On May 18,
1988, the President of the Philippines approved the same with directives that PPA shall
still have the responsibility for planning, detailed engineering, construction, expansion,
rehabilitation and capital dredging of the port, as well as the determination of how the
revenues of the port system shall be allocated for future works; and the contractor shall
not collect taxes and duties except that in the case of wharfage or tonnage dues.

ISSUE: Whether a franchise is needed for the operation of the MICT?

RULING: No. While the PPA has been tasked under E.O. No. 30 with the management
and operation of the MICT and to undertake the provision of cargo handling and port
related services thereat, the law provides that such shall be “in accordance with P.D. 857
and other applicable laws and regulations”. P.D. 857 expressly empowers the PPA to
provide services within Port Districts “whether on its own, by contract, or otherwise”. Even
if the MICT is considered a public utility, its operation would not necessarily need a
franchise from the legislature because the law has granted certain administrative
agencies the power to grant licenses for or to authorize the operation of public utilities.
Reading E.O. 30 and P.D. 857 together, it is clear that the lawmaker has empowered the
PPA to undertake by itself the operation and management of the MICP or to authorize its
operation and management by another by contract or other means, at its option.

MP: The law granted certain administrative agencies the power to grant licenses for the
operation of public utilities. Theory that MICT is a “wharf” or a “dock”, as contemplated
under the Public Service Act, would not necessarily call for a franchise from the
Legislative Branch.

CASE NO. 1290 – SABTALUH


ARTICLE XII: SEC. 11 – PUBLIC UTILITIES
TATAD VS. GARCIA

FACTS:
In 1989, DOTC planned to construct a light railway transit line along EDSA (EDSA LRT
III) to provide a mass transit system and alleviate the congestion and growing
transportation problem in the metropolis. RA 6957 was enacted allowing for the financing,
construction and operation of government projects through private initiative and
investment. EDSA LRT Corporation (organized under HK laws) was awarded with the
contract. The President approved the awarding of the contract. Petitioners questioned
that a foreign corporation cannot own the EDSA LRT III, a public utility as it violates the
Constitution.
ISSUE: Whether or not the EDSA LRT III, a public utility, can be owned by a foreign
corporation.
RULING: YES, EDSA LRT Corporation, Ltd. Is admittedly a foreign corporation “duly
incorporated and existing under the laws of Hong Kong”. However, there is no dispute
that once the EDSA LRT III is constructed, the private respondent, as lessor, will turn it
over to DOTC as lessee, for the latter to operate the system and pay rentals for the said
use. What private respondent owns are the rail tracks, rolling stocks, rail stations,
terminals and the power plant, not a public utility. While a franchise is needed to operate
these facilities to serve the public, they do not themselves constitute a public utility. What
constitutes a public utility in not their ownership but their use to serve the public. The
Constitution, in no uncertain terms, requires a franchise for the operation of a public utility.
However, it does not require a franchise before one can own the facilities needed to
operate a public utility so long as it does not operate them to serve the public. In law,
there is a clear distinction between the “operation” of a public utility and the ownership of
the facilities and equipment used to serve the public. Ownership is defined as a relation
in law by virtue of which a thing pertaining to one person is completely subjected to his
will in everything not prohibited by law or the concurrence with the rights of another. Even
the mere formation of a public utility corporation does not ipso facto characterize the
corporation as one operating a public utility. The moment for determining the requisite
Filipino nationality is when the entity applies for a franchise, certificate or any other form
of authorization for that purpose.
MAINPOINT: A foreign corporation could construct and own facilities for a light rail
transit system but it may not be given the franchise to operate the system.
CASE NO: 1291 - VARGAS
ARTICLE XII: SECTION 11 - PUBLIC UTILITIES
TELEBAP V. COMELEC

FACTS: Petitioner Telecommunications and Broadcast Attorneys of the Philippines, Inc.


(TELEBAP) is an organization of lawyers of radio and television broadcasting companies.
It was declared to be without legal standing to sue in this case as, among other reasons,
it was not able to show that it was to suffer from actual or threatened injury as a result of
the subject law. Petitioner GMA Network, on the other hand, had the requisite standing
to bring the constitutional challenge. Petitioner operates radio and television broadcast
stations in the Philippines affected by the enforcement of Section 92, B.P. No. 881.
Petitioners challenge the validity of Section 92, B.P. No. 881 which provides:
“Comelec Time- The Commission shall procure radio and television time to be known as
the “Comelec Time” which shall be allocated equally and impartially among the
candidates within the area of coverage of all radio and television stations. For this
purpose, the franchise of all radio broadcasting and television stations are hereby
amended so as to provide radio or television time, free of charge, during the period of
campaign.”
Petitioner contends that while Section 90 of the same law requires COMELEC to procure
print space in newspapers and magazines with payment, Section 92 provides that air time
shall be procured by COMELEC free of charge. Thus it contends that Section 92 singles
out radio and television stations to provide free air time.
Petitioner claims that it suffered losses running to several million pesos in providing
COMELEC Time in connection with the 1992 presidential election and 1995 senatorial
election and that it stands to suffer even more should it be required to do so again this
year. Petitioners claim that the primary source of revenue of the radio and television
stations is the sale of air time to advertisers and to require these stations to provide free
air time is to authorize unjust taking of private property. According to petitioners, in 1992
it lost P22,498,560.00 in providing free air time for one hour each day and, in this year’s
elections, it stands to lost P58,980,850.00 in view of COMELEC’s requirement that it
provide at least 30 minutes of prime time daily for such.

ISSUE: (1)Whether Section 92 of B.P. No. 881 denies radio and television broadcast
companies the equal protection of the laws.
(2) Whether Section 92 of B.P. No. 881 constitutes taking of property without due process
of law and without just compensation.

RULING: No. All broadcasting, whether radio or by television stations, is licensed by the
government. Airwave frequencies have to be allocated as there are more individuals who
want to broadcast that there are frequencies to assign. Radio and television broadcasting
companies, which are given franchises, do not own the airwaves and frequencies through
which they transmit broadcast signals and images. They are merely given the temporary
privilege to use them. Thus, such exercise of the privilege may reasonably be burdened
with the performance by the grantee of some form of public service. In granting the
privilege to operate broadcast stations and supervising radio and television stations, the
state spends considerable public funds in licensing and supervising them.

MAIN POINT: The argument that the subject law singles out radio and television stations
to provide free air time as against newspapers and magazines which require payment of
just compensation for the print space they may provide is likewise without merit.
Regulation of the broadcast industry requires spending of public funds which it does not
do in the case of print media. To require the broadcast industry to provide free air time for
COMELEC is a fair exchange for what the industry gets.

CASE NO. 1292 - TIPOE


ART. XII SEC. 11: PUBLIC UTILITIES
REPUBLIC V. EXPRESS TELECOM (EXTELCOM) CO., INC. [G.R. NO. 147096.
JANUARY 15, 2002.]

FACTS: International Communications Corporation (now Bayantel) filed an application


with the National Telecommunications Commission (NTC) for a Certificate of Public
Convenience or Necessity (CPCN) to install, operate and maintain Cellular Mobile
Telephone Service (CMTS). NTC issued memorandum circular reallocating additional
frequencies for CMTS. Respondent Express Telecommunication Co., Inc. (Extelcom)
filed its opposition against the application of Bayantel for said frequency bands and
prayed for the dismissal of the application.

ISSUE: W/N Extelcom has the right to file a petition prohibiting NTC from granting
Bayantel authority to operate CMTS and to dismiss the application.

RULING: NO. Extelcom does not enjoy the grant of any vested interest on the right to
render a public service. The Constitution is quite emphatic that the operation of a public
utility shall not be exclusive.

MAIN POINT: The Constitution mandates that a franchise cannot be exclusive in nature
nor can a franchise be granted except that it must be subject to amendment, alteration,
or even repeal by the legislature when the common good so requires. (Art. XII, Sec. 11
of the 1987 Constitution).

CASE NO. 1293 - CHIONG


ARTICLE XII: SECTION 11. PUBLIC UTILITIES
DEL MAR V. PAGCOR [2001]

FACTS:
Petitioner Raoul del Mar, a member of the House of Representatives, filed a petition for
certiorari to prevent respondent PAGCOR from managing and/or operating the jai-alai or
Basque pelota games, with Belle Jai Alai Corporation (BELLE) and Filipinas Gaming
Entertainment Totalizator Corporation (FILGAME) on the ground that the controverted act
is patently illegal and devoid of any basis either from the Constitution or PAGCORs own
Charter.

ISSUE:
Whether or not the contracts entered into by PAGCOR with its BELLE and FILGAME are
void for being undue delegations by PAGCOR of its franchise to operate and maintain
gambling casinos, sports, gaming pools and the like

RULING:
Yes, contracts entered into by PAGCOR with BELLE and FILGAME is void
PAGCOR is allowed under its charter to enter into operator‘s and/or management
contracts, it is not allowed under the same charter to relinquish or share its franchise,
much less grant a veritable franchise to another entity such as SAGE. PAGCOR cannot
delegate its power in view of the legal principle of delegata potestas delegare non potest,
inasmuch as there is nothing in the charter to show that it has been expressly authorized
to do so. In Lim v. Pacquing, the Court clarified that “since ADC has no franchise from
Congress to operate the jai-alai, it may not so operate even if it has a license
or permit from the City Mayor to operate the jai-alai in the City of Manila.” By the same
token, SAGE has to obtain a separate legislative franchise and not “ride on” PAGCOR‘s
franchise if it were to legally operate on-line Internet gambling.
MAIN POINT:
PAGCOR has a valid franchise to, but only by itself (i.e., not in association with any other
person or entity), operate, maintain and/or manage the game of jai-alai, and to DENY the
motions insofar as respondents would also seek a reconsideration of the Court’s decision
of 29 November 2000 that has, since then, enjoined the continued operation,
maintenance, and/or management of jai-alai games by PAGCOR in association with its
co- respondents Belle Jai-Alai Corporation and/or Filipinas Gaming Entertainment
Totalizator Corporation and held to be without force and effect the agreement of 17 June
1999 among said respondents.

CASE NO. 1294 - LAKBAO


Article XII. National Economy and Patrimony. Section 11. Public Utilities
PTC v. NTC
Facts:
Issue:
Ruling:

MP:
CASE No. 1295 – SABTALUH
ARTICLE XII: SEC. 11 – PUBLIC UTILITIES
ASSOCIATED COMMUNICATIONS & WIRELESS SERVICES – UNITED
BROADCASTING vs. NTC

FACTS: On November 11, 1931, Act No. 3846, entitled "An Act Providing for the
Regulation of Radio Stations and Radio Communications in the Philippines and for Other
Purposes," was enacted. Sec. 1 of the law reads, "Sec. 1. No person, firm, company,
association, or corporation shall construct, install, establish, or operate a radio
transmitting station, or a radio receiving station used for commercial purposes, or a radio
broadcasting station, without having first obtained a franchise therefor from the Congress
of the Philippines..."
in 1969 Marcos J. Villa Verde, Jr. and Winfred E. Villaverde transferred their 50-year
franchise to Construct, Install, maintain and Operate Public Radio telephone and
Radiotelegraph Coastal Stations, and Public Fixed and Public Based and Land Mobile
Stations to Associated Communications & Wireless Services – United Broadcasting
Network, Inc. (ACWS for brevity) through Congress’ Concurrent Resolution No. 58.
Petitioner ACWS then engaged in the installation and operation of several radio stations
around the country. in 1979, E.O. No. 5464 was issued. It integrated the Board of
Communications and the Telecommunications Control Bureau under the Integrated
Reorganization Plan of 1972 into the NTC. Among the powers vested in the NTC under
Sec. 15 of E.O. No. 546 are the following: a. Issue Certificate of Public Convenience for
the operation of communication utilities and service. b.) Grant permits for the use of radio
frequencies for wireless telephone and telegraph systems and radio communication
system. Upon termination of petitioner’s franchise on December 31, 1981 pursuant to
P.D. No. 576-A, it continued operating its radio stations under permits granted by the
NTC.
ISSUE: Whether or not the operation of a radio or television station requires a
congressional franchise.

RULING: No, NTC is empowered under E.O. No. 546 to issue authorization and permits
to operate radio and television broadcasting system.
The Supreme Court stated that the provision in the Constitution (Art. XII, Sec. 11) 'that
the issuance of a franchise, certificate or other form of authorization for the operation of
a public utility shall be subject to amendment, alteration or repeal by Congress, does not
necessarily imply. That only Congress has the power to grant such authorization. Our
statute books are replete with laws granting specified agencies in the Executive Branch
the power to issue such authorization for certain classes of public utilities.

MAINPOINT: The power to issue franchise is legislative in nature but delegable.


CASE NO: 1296 - VARGAS
ARTICLE XII: SECTION 11. PUBLIC UTILITIES
EASTERN TELECOM V. TELECOM TECHNOLOGIES

FACTS: The role of the telecommunications industry in Philippine progress and


development cannot be understated. Time was when the industry was dominated by a
few -- an oligarchy of sorts where the elite made the decisions and serfdom had no choice
but acquiesce. Sensing the need to abrogate their dominion, the government formulated
policies in order to create an environment conducive to the entry of new players. Thus,
in October 1990, the National Telecommunications Development Plan 1991-2010
(NTDP) was formulated and came into being. Designed by the Department of
Transportation and Communications (DOTC), the NTDP provides for the framework of
government policies, objectives and strategies that will guide the industry's development
for the next 20 years. As expected, with it came the increase in the demand for
telecommunications services, especially in the area of local exchange carrier service
(LECS).

ISSUE: Whether or not the Honorable Court of Appeals committed a serious error of law
in upholding the Order of the NTC granting a PA to Respondent to operate LEC services
in Manila and Navotas which are areas already assigned to petitioner TTPI under a prior
and subsisting PA.

RULING: After a review of the records of this case, the Court finds no grave abuse of
discretion committed by the Court of Appeals in sustaining the NTC's grant of provisional
authority to ICC.
The power of the NTC to grant a provisional authority has long been settled. As the
regulatory agency of the national government with jurisdiction over all
telecommunications entities, it is clothed with authority and given ample discretion to
grant a provisional permit or authority. It also has the authority to issue Certificates of
Public Convenience and Necessity (CPCN) for the installation, operation, and
maintenance of communications facilities and services, radio communications systems,
telephone and telegraph systems, including the authority to determine the areas of
operations of applicants for telecommunications services. In this regard, the NTC is
clothed with sufficient discretion to act on matters solely within its competence.

CASE NO. 1297 - TIPOE


ART. XII SEC. 11: PUBLIC UTILITIES
ROYAL CARGO CORPORATION V. CIVIL AERONAUTICS BOARD (CAB) [G.R.
NOS. 103055-56. JANUARY 26, 2004.]

FACTS: The petitioner Royal Cargo Corporation is a stock corporation duly organized
and existing under and by virtue of Philippine laws, seventy percent (70%) of which is
owned by Filipino citizens and thirty percent (30%) by foreigners. The President of the
petitioner company is a foreigner who is married to a Filipina.
ISSUE: W/N the CAB can require the petitioner to transfer the position of President to a
Filipino national.

RULING: YES. Under the Constitution, such position must be held by a Filipino national.

MAIN POINT: ART. XII Sec 11 requires: “. . . The participation of foreign investors in the
governing body of any public utility enterprise shall be limited to their proportionate share
in its capital, and all the executive and managing officers of such corporation or
association must be citizens of the Philippines.”

CASE NO. 1298 - CHIONG


ARTICLE XII: SECTION 11. PUBLIC UTILITIES
METROPOLITAN CEBU WATER DISTRICT (MCWD) V. M. ADALA {526 SCRA 465}
FACTS:
The Metropolitan Cebu Water District (MCWD), a public corporation, appealed the
decision rendered in favor of Margarita A. Adala (Adala) by the National Water Resources
Board (NWRB), granting her a franchise permit to supply water to three sitios in Bulacao.
MCWD was the exclusive distributor of water in the district. MCWD contended that the
proposed waterworks would interfere with their water supply which it has the right to
protect, and the water needs of the residents in the subject area was already being well
served by petitioner. They also contend that they were granted by Section 47 of
Presidential Decree 198, granting exclusive franchise only to public utilities. Engineer
Paredes, the general manager of MCWD, filed Certificate of Public Convenience by the
National Water Resources Board (NWRB), which permitted the company to operate and
maintain waterworks supply services. MCWD alleged that the Board of Directors of
MCWD did not give consent to the issuance of the franchise applied for.

ISSUES: Whether or not Section 47 of Presidential Decree 198 grants exclusive


franchise to public utilities

RULING: MWCD‘s position that an overly strict construction of the term ―franchise as
used in Section 47 of P.D. 198 would lead to an absurd result impresses. If franchises, in
this context, were strictly understood to mean an authorization issuing directly from the
legislature, it would follow that, while Congress cannot issue franchises for operating
waterworks systems without the water district ‘s consent, the NWRB may keep on issuing
CPCs authorizing the very same acteven without such consent. In effect, not only would
the NWRB be subject to less constraints than Congress in issuing franchises. The
exclusive character of the franchise provided for by Section 47 would be illusory. While
the prohibition in Section 47 of P.D. 198 applies to the issuance of CPCs for the reasons
discussed above, the same provision must be deemed void ab initio for being
irreconcilable with Article XIV Section 5 of the 1973 Constitution which was ratified on
January 17, 1973 – the constitution in force when P.D. 198 was issued on May 25, 1973.
That the legislative authority – in this instance, then President Marcos – intended to
delegate its power to issue franchises in the case of water districts is clear from the fact
that, pursuant to the procedure outlined in P.D. 198, it no longer plays a direct role in
authorizing the formation and maintenance of water districts, it having vested the same
to local legislative bodies and the Local Water Utilities Administration (LWUA).

MAIN POINT:

The term “franchise” broadly so as to include, not only authorizations issuing directly from
Congress in the form of statute, but also those granted by administrative agencies to
which the power to grant franchises has been delegated by Congress.

CASE NO. 1299 - LAKBAO


ARTICLE XII: SECTION 11. PUBLIC UTILITIES
PAGCOR V. BIR

FACTS: Petitioner further seeks to prohibit the implementation of Bureau of Internal


Revenue (BIR) Revenue Regulations No. 16-2005 for being contrary to law. With the
enactment of R.A. No. 9337[10] on May 24, 2005, certain sections of the National Internal
Revenue Code of 1997 were amended. Different groups came to this Court via petitions
for certiorari and prohibition[11] assailing the validity and constitutionality of R.A. No. 9337
10% Value Added Tax (VAT) on sale of goods and properties 10% VAT on importation of
goods 10% VAT on sale of services and use or lease of properties... the Court dismissed
all the petitions and upheld the constitutionality of R.A. No. 9337. On the same date,
respondent BIR issued Revenue Regulations (RR) No. 16-2005,[13] specifically
identifying PAGCOR as one of the franchisees subject to 10% VAT imposed under
Section 108 of the National Internal Revenue Code of 1997, as amended by R.A. No.
9337. Furthermore, according to the OSG,... public respondent BIR exceeded its statutory
authority when it enacted RR No. 16-2005, because the latter's provisions are contrary to
the mandates of P.D. No. 1869 in relation to R.A. No. 9337.

ISSUE: Is Republic Act 9337 constitutional insofar as it excluded PAGCOR from the
enumeration of GOCCs exempt from the payment of corporate income tax?

RULING: YES. The original exemption of PAGCOR from corporate income tax was not
made pursuant to a valid classification based on substantial distinctions so that the law
may operate only on some and not on all. The Court holds that the provision subjecting
PAGCOR to 10% VAT is invalid for being contrary to R.A. No. 9337. Nowhere in R.A.
No. 9337 is it provided that petitioner can be subjected to VAT. R.A. No. 9337 is clear
only as to... the removal of petitioner's exemption from the payment of corporate income
tax, which was already addressed above by this Court. Instead, the same was merely
granted due to the acquiescence of the House Committee on Ways and Means to the
request of PAGCOR. The argument that the withdrawal of the exemption also violates the
non-impairment clause will not hold since any franchise is subject to amendment,
alteration or repeal by Congress. However, the Court made it clear that PAGCOR remains
exempt from payment of indirect taxes and as such its purchases remain not subject to
VAT, reiterating the rule laid down in the Acesite case.

MP: The following services performed in the Philippines by VAT-registered persons shall
be subject to zero percent (0%) rate; Services rendered to persons or entities whose
exemption under special laws... subjects the supply of such services to zero percent (0%)
rate... although R.A. No. 9337 introduced amendments to Section 108 of R.A. No. 8424
by imposing VAT on other services not previously covered, it did not amend the portion
of Section 108 (B) (3) that subjects to zero percent rate services performed by VAT-
registered persons to persons or entities whose exemption under special laws or
international agreements to which the Philippines is a signatory effectively subjects the
supply of such services to 0% rate.

CASE NO: 1301 - VARGAS


ARTICLE XII: SECTION 11-PUBLIC UTILITIES
WILSON P. GAMBOA V. FINANCE SECRETARY MALGARITO B TEBES

FACTS: This is a petition to nullify the sale of shares of stock of Philippine


Telecommunications Investment Corporation (PTIC) by the government of the Republic
of the Philippines, acting through the Inter-Agency Privatization Council (IPC), to Metro
Pacific Assets Holdings, Inc. (MPAH), an affiliate of First Pacific Company Limited (First
Pacific), a Hong Kong-based investment management and holding company and a
shareholder of the Philippine Long Distance Telephone Company (PLDT).
The petitioner questioned the sale on the ground that it also involved an indirect sale of
12 million shares (or about 6.3 percent of the outstanding common shares) of PLDT
owned by PTIC to First Pacific. With the this sale, First Pacific’s common shareholdings
in PLDT increased from 30.7 percent to 37 percent, thereby increasing the total common
shareholdings of foreigners in PLDT to about 81.47%. This, according to the petitioner,
violates Section 11, Article XII of the 1987 Philippine Constitution which limits foreign
ownership of the capital of a public utility to not more than 40%

ISSUE: Whether the term “capital” in Section 11, Article XII of the Constitution refer to the
total common shares only, or to the total outstanding capital stock (combined total of
common and non-voting preferred shares) of PLDT, a public utility?

RULING: It must be stressed, and respondents do not dispute, that foreigners hold a
majority of the common shares of PLDT. In fact, based on PLDT’s 2010 General
Information Sheet (GIS), which is a document required to be submitted annually to the
Securities and Exchange Commission, foreigners hold 120,046,690 common shares of
PLDT whereas Filipinos hold only 66,750,622 common shares. In other words, foreigners
hold 64.27% of the total number of PLDT’s common shares, while Filipinos hold only
35.73%. Since holding a majority of the common shares equates to control, it is clear that
foreigners exercise control over PLDT. Such amount of control unmistakably exceeds the
allowable 40 percent limit on foreign ownership of public utilities expressly mandated in
Section 11, Article XII of the Constitution.

MAIN POINT: In short, Filipinos hold less than 60 percent of the voting stock, and earn
less than 60 percent of the dividends, of PLDT. This directly contravenes the express
command in Section 11, Article XII of the Constitution that “[n]o franchise, certificate, or
any other form of authorization for the operation of a public utility shall be granted except
to corporations organized under the laws of the Philippines, at least sixty per centum of
whose capital is owned by such citizens.

Case No. 1302 - TIPOE


ART. XII Sec. 11: PUBLIC UTILITIES
EXPRESS INVESTMENTS III PRIVATE LTD. v. BAYAN TELECOM (BAYANTEL),
INC. [G.R. Nos. 174457-59. December 5, 2012.]

FACTS: On various dates between the years 1995 and 2001, Bayantel entered into
several credit agreements. Foreseeing the impossibility of further meeting its obligations,
Bayantel sent a proposal for the restructuring of its debt. Ultimately, the corporation was
issued a stop order for the suspension of all claims and was placed under Rehabilitation
by the court. Petitioners maintain that converting the unsustainable debt to 77.7% equity
in Bayantel will not violate the nationality requirement of the 1987 Constitution

ISSUE: W/N the conversion of debt to equity in excess of 40% of the outstanding capital
stock in favor of petitioners violates the constitutional limit on foreign ownership of a public
utility.

RULING: YES. In its Rehabilitation Plan, respondent Bayantel shall "relinquish the
agreed-upon amount of common stock[s] as payment to Unsecured Creditors as per the
Term Sheet." Evidently, the parties intend to convert the unsustainable portion of
respondent's debt into common stocks, which have voting rights. If we indulge petitioners
on their proposal, the Omnibus Creditors which are foreign corporations, shall have
control over 77.7% of Bayantel, a public utility company. This is precisely the scenario
proscribed by the Filipinization provision of the Constitution. Therefore, the Court of
Appeals acted correctly in sustaining the 40% debt-to-equity ceiling on conversion.

MAIN POINT: Art. XII Sec. 11 of the Constitution explicitly reserves to Filipino citizens
control over public utilities, pursuant to an overriding economic goal of the 1987
Constitution: to "conserve and develop our patrimony" and ensure "a self-reliant and
independent national economy effectively controlled by Filipinos."
CASE NO. 1303 - CHIONG
ARTICLE XII: SECTION 11. PUBLIC UTILITIES
JG SUMMIT HOLDINGS VS CA
FACTS:
Kawasaki Heavy Industries, Ltd., a Japanese corporation, entered into a joint venture with
the government's National Investment and Development Corporation (NIDC) for the
construction, operation and management of a public utility, the Subic National Shipyard,
Inc, now Philippine Shipyard and Engineering Corporation (PHILSECO), with the parties
given the right of first refusal. The government's share in PHILSECO was increased to
97.41% leaving Kawasaki's share to 2.59%. The APT, as trustee for the government,
offered for sale its 87.67% share in PHILSECO giving KAWASAKI the right to top by 5%
the highest bid in "exchange" for its right of first refusal and can designate a company
which could exercise said right. The consortium, composed of petitioner and two other
foreign corporations, was declared the highest bidder at P2.03 billion. Nonetheless,
KAWASAKI's designated company, PHI consortium, paid the purchase price and a stock
purchase agreement thereafter issued in its favor. Petitioner challenged the same by a
petition for mandamus with the Court of Appeals. The appellate court, in ruling that the
right of first refusal and the right to top are prima facie legal and that petitioner is in
estoppel, dismissed the petition. Hence, this recourse of petitioner.

ISSUE:
Whether or not PHILSECO is a public utility.

RULING:
Under the Constitution, the operation of shipbuilding and ship repair industry is a public
utility which may be granted only to Filipino citizens or associations organized under
Philippine laws at least 60% of whose capital is owned by such citizens. The participation
of foreign investors is limited to their proportionate maximum share of 40%. Allowing a
foreign corporation to participate in a public bidding in the sale of a public utility beyond
its authorized share is unconstitutional.

MAIN POINT:
It is crystal clear that a shipyard cannot be considered a public utility. A shipyard is a place
or enclosure where ships are built or repaired. Its nature dictates that it serves but a
limited clientele whom it may choose to serve at its discretion. While it offers its facilities
to whoever may wish to avail of its services, a shipyard is not legally obliged to render its
services indiscriminately to the public. It has no legal obligation to render the services
sought by each and every client. The fact that it publicly offers its services does not give
the public a legal right to demand that such services be rendered.

CASE NO. 1304 - LAKBAO


ARTICLE XII: SECTION 12- FILIPINO FIRST POLICY
TANADA V. ANGARA
FACTS: Petitioners assail the constitutionality of the Philippines acceding to the World
Trade Organization for being violative of provisions which are supposed to give
preference to Filipino workers and economy and on the ground that it infringes legislative
and judicial power. The
WTO, through it provisions on “most favored nation”and national treatment, require that
nationals and other member countries are placed in the same footing in terms of products
and services. However, the Court brushed off these contentions and ruled that the WTO
is constitutional. Sections 10 and 12 of Article XII (National Economy and Patrimony)
should be read in relation to Sections 1 and 13 (promoting the general welfare). Also,
Section 10 is self-executing only to “rights, privileges, and concessions covering national
economy and patrimony” but
not every aspect of trade and commerce. There are balancing provisions in the
Constitution allowing the Senate to ratify the WTO agreement. Also, the Constitution
doesn’t rule out foreign competition. States waive certain amount of sovereignty when
entering into treaties.

ISSUE: Whether the provision of WTO contravenes Section 10 & 12, Artilce XII of the
1987 Constitution?

RULING: No. Secs. 10 and 12 of Article XII should be read and understood in relation to
the other sections in said article, especially Sec. 1 and 13: (1) A more equitable
distribution of opportunities, income and wealth; (2) A sustained increase in the amount
of goods and services; (3) An expanding productivity as the key to raising the quality of
life. The issue here is not whether this paragraph of Sec. 10 of Art. XII is self-executing
or not.Rather, the issue is whether, as a rule, there are enough balancing provisions in
the Constitution to allow the Senate to ratify the Philippine concurrence in the WTO
Agreement. And we hold that there are WTO Recognizes Need to Protect Weak
Economies. Unlike in the UN where major states have permanent seats and veto powers
in the Security Council, in the WTO, decisions are made on the basis of sovereign
equality, with each member’s vote equal in weight. Specific WTO Provisos Protect
Developing Countries
Tariff reduction – developed countries must reduce at rate of 36% in 6 years, developing
24% in 10 years
Domestic subsidy – developed countries must reduce 20% over six (6) years, developing
countries at 13% in 10 years
Export subsidy– developed countries, 36% in 6 years; developing countries, 3/4ths of
36% in 10 years

MP: Constitution Does Not Rule Out Foreign Competition. Encourages industries that are
competitive in both domestic and foreign markets. The Court will not pass upon the
advantages and disadvantages of trade liberalization as an economic policy. It will only
perform its constitutional duty of determining whether the Senate committed grave abuse
of discretion.
CASE NO. 1305 – SABTALUH
ARTICLE XII: SEC. 13 -TRADE POLICY
ESPINA V. ZAMORA

FACTS: On March 7, 2000 Republic Act (R.A.) 8762, also known as the Retail Trade
Liberalization Act of 2000 was enacted into law. It expressly repealed R.A. 1180, which
absolutely prohibited foreign nationals from engaging in the retail trade business. R.A.
8762 now allows them to do so under four categories:
Category A Less thanUS$2,500,000.00Exclusively for Filipino citizens and corporations
wholly owned by Filipino citizens. Category B US$2,500,000.00 up but less
thanUS$7,500,000.00 For the first two years of R.A. 8762’s effectivity, foreign ownership
is allowed up to 60%. After the two-year period, 100% foreign equity shall beallowed.
Category C US$7,500,000.00 or more May be wholly owned by foreigners. Foreign
investments for establishing a store in Categories B and C shall not be less than the
equivalent in Philippine Pesos of US$830,000.00. Category D US$250,000.00 per store
of foreign enterprises specializing in high-end or luxury products May be wholly owned by
foreigners.R.A. 8762 also allows natural-born Filipino citizens, who had lost their
citizenship and now reside in the Philippines, to engage in the retail trade business with
the same rights as Filipino citizens. On October 11, 2000, petitioners assailed that RA
8762 breach the constitutional mandate for the development of a self-reliant and
independent national economy effectively controlled by Filipinos

ISSUE: Whether or not R.A. 8762 is unconstitutional.

RULING: No, While Section 19, Article II of the 1987 Constitution requires the
development of a self-reliant and independent national economy effectively controlled by
Filipino entrepreneurs, it does not impose a policy of Filipino monopoly of the economic
environment. The objective is simply to prohibit foreign powers or interests from
maneuvering our economic policies. Section 10, Article XII of the 1987 Constitution gives
Congress the discretion to reserve to Filipinos certain areas of investments upon the
recommendation of the National Economic and Development Authority and when the
national interest requires. Thus, Congress can determine what policy to pass and when
to pass it depending on the economic exigencies. It can enact laws allowing the entry of
foreigners into certain industries not reserved by the Constitution to Filipino citizens. In
this case, Congress has decided to open certain areas of the retail trade business to
foreign investments instead of reserving them exclusively to Filipino citizens.

MAINPOINT: Foreigners can enter into certain industries not reserved by the Constitution
to Filipino citizens.
CASE NO: 1306 - VARGAS
ARTICLE XII: SECTION 16. CORPORATIONS
NDC V. PVB

Facts: The particular enactment in question is Presidential Decree No. 1717, which
ordered the rehabilitation of the Agrix Group of Companies to be administered mainly by
the National Development Company. The law outlined the procedure for filling claims
against the Agrix Companies and created a claims committee to process these claims.
Especially relevant to this case, and noted at the outset, is section 4(1) thereof providing
that “all mortgages and other liens presently attaching to any of the assets of the dissolved
corporations are hereby extinguished.” Earlier, the Agrix Marketing Inc. had executed in
favor of private respondent Philippine Veterans Bank a real estate mortgage dated July
7, 1978 over three parcels of land situated in Los Baños, Laguna. During the existence
of the mortgage, Agrix went bankrupt. It was the expressed purpose of salvaging this and
the other Agrix companies that the aforementioned decree was issued by President
Marcos. A claim for the payment of its loan credit was filed by PNB against herein
petitioner, however the latter alleged and invoked that the same was extinguished by PD
1717.

Issue: Whether or not Philippine Veterans Bank as creditor of Agrix is still entitled for
payment without prejudice to PD 1717.

Ruling: Yes. A mortgage lien is a property right derived from contract and so comes
under the protection of Bill of rights so do interests on loans, as well as penalties and
charges, which are also vested rights once they accrue. Private property cannot simply
be taken by law from one person and given to another without just compensation and any
known public purpose. This is plain arbitrariness and is not permitted under the
constitution.

Main Point: PD 1717 is an invalid exercise of the police power, not being in conformity
with the traditional requirements of a lawful subject and a lawful method. The extinction
of the mortgage and other liens and of the interest and other charges pertaining to the
legitimate creditors of Agrix constitutes taking without due process of law, and this is
compounded by the reduction of the secured creditors to the category of unsecured
creditors in violation of the equal protection clause.
CASE NO. 1307 - TIPOE
ARTICLE XII: SEC. 16- CORPORATIONS
BOY SCOUTS OF THE PHILIPPINES (BSP) V. COA

FACTS: This case arose when the Commission on Audit (COA) issued a resolution
stating that the BSP was created as a public corporation under Commonwealth Act No.
111, as amended. And hence, it is subject to audit by COA. The COA maintains that the
functions of the BSP are undeniably sovereign functions enshrined under the Constitution
The COA contends that any attempt to classify the BSP as a private corporation would
be incomprehensible since no less than the law which created it had designated it as a
public corporation and its statutory mandate embraces performance of sovereign
functions.

ISSUE: W/N the creation of BSP through a special law violates the constitutional provision
Art. XII Sec. 16.

RULING: NO. Article XII, Section 16 bans the creation of "private corporations" by special
law. The said constitutional provision should not be construed so as to prohibit the
creation of public corporations or a corporate agency or instrumentality of the government
intended to serve a public interest or purpose

MAIN POINT: Art. XII Sec. 16 of the Constitution should not be construed so as to prohibit
Congress from creating public corporations. In fact, Congress has enacted numerous
laws creating public corporations or government agencies or instrumentalities vested with
corporate powers. Moreover, said provision could not have tied the hands of Congress in
creating public corporation to serve any of the constitutional policies or objective.

CASE NO. 1308 - CHIONG


ARTICLE XII: SECTION 17 - TEMPORARY TAKE-OVER
AGAN V. PIATCO

FACTS:
On October 5, 1994, AEDC submitted an unsolicited proposal to the Government through
the DOTC/MIAA for the development of NAIA International Passenger Terminal III (NAIA
IPT III).

DOTC constituted the Prequalification Bids and Awards Committee (PBAC) for the
implementation of the project and submitted with its endorsement proposal to the NEDA,
which approved the project. On June 7, 14, and 21, 1996, DOTC/MIAA caused the
publication in two daily newspapers of an invitation for competitive or comparative
proposals on AEDC’s unsolicited proposal, in accordance with Sec. 4-A of RA 6957, as
amended. On September 20, 1996, the consortium composed of People’s Air Cargo and
Warehousing Co., Inc. (Paircargo), Phil. Air and Grounds Services, Inc. (PAGS) and
Security Bank Corp. (Security Bank) (collectively, Paircargo Consortium) submitted their
competitive proposal to the PBAC. PBAC awarded the project to Paircargo Consortium.
Because of that, it was incorporated into Philippine International Airport Terminals Co.,
Inc. AEDC subsequently protested the alleged undue preference given to PIATCO and
reiterated its objections as regards the prequalification of PIATCO.

ISSUE:
Whether or not the State can temporarily take over a business affected with public
interest.

RULING:
Yes. PIATCO cannot, by mere contractual stipulation, contravene the Constitutional
provision on temporary government takeover and obligate the government to pay
“reasonable cost for the use of the Terminal and/or Terminal Complex.” Article XII, section
17 of the 1987 Constitution envisions a situation wherein the exigencies of the times
necessitate the government to “temporarily take over or direct the operation of any
privately owned public utility or business affected with public interest.” It is the welfare
and interest of the public which is the paramount consideration in determining whether or
not to temporarily take over a particular business. Clearly, the State in effecting the
temporary takeover is exercising its police power. Police power is the “most essential,
insistent, and illimitable of powers.” Its exercise therefore must not be unreasonably
hampered nor its exercise be a source of obligation by the government in the absence of
damage due to arbitrariness of its exercise. Thus, requiring the government to pay
reasonable compensation for the reasonable use of the property pursuant to the operation
of the business contravenes the Constitution

MAIN POINT: The nature and extent of the emergency is the measure of the duration of
the takeover as well as the terms thereof. It is the State that prescribes such reasonable
terms which will guide the implementation of the temporary takeover as dictated by the
exigencies of the time.
CASE NO. 1309 - LAKBAO
ARTICLE XII: SECTION 17. TEMPORARY TAKE- OVER
DAVID V. MACAPAGAL-ARROYO

FACTS: On February 24, 2006, as the nation celebrated the 20thAnniversary of the
EDSA PeoplePower I, President Gloria Macapagal-Arroyo, in a move to suppress alleged
plans to overthrow thegovernment, issued Presidential Proclamation No. 1017 (PP 1017),
declaring a state of nationalemergency. She cited as factual bases for the said issuance
the escape of the Magdalo Group andtheir audacious threat of the Magdalo D-Day; the
defections in the military, particularly in thePhilippine Marines; and the reproving
statements from the communist leaders. On the same day, sheissued General Order No.
5 (G.O. No. 5) setting the standards which the Armed Forces of thePhilippines (AFP) and
the Philippine National Police (PNP) should follow in the suppression andprevention of
acts of lawless violence. The following were considered as additional factual bases forthe
issuance of PP 1017 and G.O. No. 5: the bombing of telecommunication towers and cell
sites in Bulacan and Bataan; the raid of an army outpost in Benguet resulting in the death
of three soldiers; and the directive of the Communist Party of the Philippines ordering its
front organizations to join5,000 Metro Manila radicals and 25,000 more from the provinces
in mass protests. Immediately, the Office of the President announced the cancellation of
all programs and activities related to the 20thPeople Power I anniversary celebration. It
revoked permits to hold rallies. Members of the Kilusang Mayo Uno (KMU) and the
National Federation of Labor Unions-Kilusang Mayo Uno (NAFLU-KMU), who marched
from various parts of Metro Manila to converge at theEDSA Shrine, were violently
dispersed by anti-riot police. Professor Randolf David, Akbayan party-list president
Ronald Llamas, and members of the KMU and NAFLU-KMU were arrested without a
warrant. In the early morning of February 25, 2006, operatives of the Criminal
Investigation and Detection Group (CIDG) raided theDaily Tribune offices in Manila and
confiscated news stories, documents, pictures, and mock-ups of the Saturday issue.
Policemen were stationed inside theeditorial and business offices, as well as outside the
building. A few minutes after the search andseizure at theDaily Tribune offices, the police
surrounded the premises of another pro-oppositionpaper, Malaya , and its sister
publication, the tabloid Abante . The PNP warned that it would take overany media
organization that would not follow “standards set by the government during the state of
national emergency.”On March 3, 2006, exactly one week from the declaration of a state
of national emergency and after all the present petitions had been filed, President Arroyo
issued Presidential ProclamationNo. 1021 (PP 1021), declaring that the state of national
emergency has ceased to exist and lifting PP1017. These consolidated petitions for
certiorari and prohibition allege that in issuing PP 1017 andG.O. No. 5, President Arroyo
committed grave abuse of discretion. It is contended that respondent officials of the
Government, in their professed efforts to defend and preserve democratic institutions, are
actually trampling upon the very freedom guaranteed and protected by the Constitution.
Hence, such issuances are void for being unconstitutional.

ISSUE: Whether or not PP 1017 authorizes the President to take over privately-owned
public utility or business affected with public interest.
RULING: PP 1017 does not authorize President Arroyo during the emergency to
temporarily take over or direct the operation of any privately owned public utility or
business affected with public interest without authority from Congress. Generally,
Congress is the repository of emergency powers. However, knowing that during grave
emergencies, it may not be possible or practicable for Congress to meet and exercise its
powers, the framers of our Constitution deemed it wise to allow Congress to grant
emergency powers to the President, subject to certain conditions, thus: (a)there must be
a war or other emergency; (b)the delegation must be for a limited period only; (c)the
delegation must be subject to such restrictions as the Congress may prescribe; and (d)the
emergency powers must be exercised to carry out a national policy declared by Congress.
The taking over of private business affected with public interest is just another facet of the
emergency powers generally reposed upon Congress. Thus, when Sec. 17, Art. XII of the
Constitution states that the “ the State may, during the emergency and under reasonable
terms prescribed by it, temporarily take over or direct the operation of any privately owned
public utility or business affected with public interest ,” it refers to Congress, not the
President. Whether or not the President may exercise such power is dependent on
whether Congress may delegate it to her pursuant to a law prescribing the reasonable
terms thereof.

MP: There is a distinction between the President’s authority to declare a state of national
emergency and her authority to exercise emergency powers. Her authority to declare a
state of national emergency is granted by Sec. 18, Art. VII of the Constitution, hence, no
legitimate constitutional objection can be raised. The exercise of emergency powers, such
as the taking over of privately owned public utility or business affected with public interest,
is a different matter. This requires a delegation from Congress. The President cannot
decide whether exceptional circumstances exist warranting the takeover of privately-
owned public utility or business affected with public interest. Nor can she determine when
such exceptional circumstances have ceased. Likewise, without legislation, the President
has no power to point out the types of businesses affected with public interest that should
be taken over.

CASE NO. 1310 - SABTALUH


ARTICLE XII: SEC. 18 - NATIONALIZATION
REPUBLIC VS PLDT {26 SCRA 620 1968}

FACTS: PLDT and RCA Communications Inc. entered into an agreement where
telephone messages, coming from the US and received by RCA's domestic station could
automatically be transferred to the lines of PLDT and vice versa. The Bureau of
Telecommunications set up its own Government Telephone System (GTS) by renting the
trunk lines of PLDT to enable government offices to call private parties. One of the many
rules prohibits the use of the service for his private use. Republic of the Philippines
entered into an agreement with RCA for a joint overseas telephone service where the
Bureau would convey radio-telephone overseas calls received by the RCA's station to
and from local residents. PLDT complained that the Bureau was violating the conditions
for using the trunk lines not only for the use of government offices but even to serve
private persons or the general public. PLDT gave a notice that if violations were not
stopped, PLDT would sever the connections -which PLDT did. Republic sued PLDT
commanding PLDT to execute a contract, through the Bureau, for the use of the facilities
of defendant's telephone system throughout the Philippines under such terms and
conditions as the court finds it reasonable.

ISSUE: Whether or Not PLDT may be compelled by the state to enter into such
agreement.

RULING: Yes, the state may, in the interest of national welfare, transfer utilities to public
ownership upon payment of just compensation, there is no reason why the state may not
require a public utility to render services in the general interest provided just
compensation is paid.

MAINPOINT: Under section 18, of Article 12, the state may compel a public utility to
render public service provided just compensation is paid for.
CASE NO: 1311 - VARGAS
ARTICLE XII: SECTION 18 - NATIONALIZATION
PLDT V. NTC

FACTS: On June 22, 1958, Republic Act No. 2090, was enacted otherwise known as “An
Act Granting Felix Alberto and Company, Incorporated, a franchise to establish radio
stations for domestic and transoceanic telecommunications.” Felix Alberto & Co. Inc. was
the original corporate name, which was changed to ETCI with amendment of the articles
of incorporation in 1964. Much later, “CELLCOM Inc.” was the name sought to be adopted
before the Securities and Exchange Commission, but this was withdrawn and
abandoned.On May 13, 1987, alleging urgent public need, ETCI filed an application with
public respondent NTC for the issuance of a certificate of public convenience and
necessity to construct, install, establish, operate, and maintain a cellular mobile telephone
system and an alpha numeric paging system in Metro Manila and in the Southern Luzon
regions, with prayer for provisional authority to operate phase A of its proposal within
Metro Manila. PLDT filed an opposition with motion to dismiss, however NTC over ruled
it. NTC granted ETC provisional authority to install, operate, and maintain a cellular
mobile telephone system initially in Metro Manila subject to terms and conditions, one of
which is that ETCI and PLDT shall enter into an interconnection agreement for the
provision of adequate interconnection facilities between applicant’s cellular mobile
telephone switch and the public switched telephone network and shall jointly submit such
interconnection agreement to the commission for approval ETCI admits that in 1964, the
Albertos, as original owners of more than 40% of the outstanding capital stock sold their
holdings to Orbes. In 1968, the Albertos reacquired the shares they had sold to the Orbes.
In 1987, the Albertos sold more than 40% of their shares to Horacio Yalung. Thereafter,
the present stockholders acquired their ETCI shares. Moreover, in 1964, ETCI had
increased its capital stock from Php40, 000 to Php360, 000; and in 1987, from Php360,
000 to Php40, 000,000.
ISSUE: Whether or not the transfers in 1987 of the shares of stock to the new
stockholder’s amount to a transfer of ETCI’s franchise which needs congressional
approval pursuant to RA 2090.

RULING: No. Section 10 of RA 2090 is directed to the grantee of the franchise, which is
the corporation itself and refers to a sale, lease or assignment of that franchise. It does
not include the transfer or sale of shares of stock of a corporation by the latter’s
stockholders.
The sale of shares of stock of a public utility is governed by another law, in section 20 (h)
of the Public Service Act (CA 146). Pursuant thereto, the public service commission (now
NTC) is the government agency vested with the authority to approve the transfer of more
than 40% of the subscribed capital stock of a telecommunications company to a single
transferee.

MAIN POINT: What transpired in ETCI were a series of transfers of shares starting in
1964 until 1987. The approval of the NTC may be deemed to have been met when it
authorized the issuance of the provisional authority to ETCI. There was full disclosure
before the NTC of the transfers.

CASE NO. 1312 - TIPOE


ART. XII SEC. 18: NATIONALIZATION
PLDT V. EASTERN TELECOM

FACTS: Eastern Telecommunications Philippines, Inc. (Eastern) filed with the NTC an
application to construct, maintain and operate an International Digital Gateway Facility
(IDGF). The gateway facility will permit messages originating from a person using PLDT's
domestic telephone system to enter the transmitting and carrying facilities of Eastern, as
well as messages incoming from abroad through Eastern's carrying facilities to enter
PLDT's domestic system. Thus, there is a need to interconnect with PLDT's domestic
telephone network as ordered by the NTC. Eastern's expounded services would be
useless if it cannot mainly use the existing PLDT grid.

ISSUE: W/N Eastern should be allowed to interconnect with PLDT to maximize the use
of telecommunications facilities available in recognition of the vital role of communications
in nation building.

RULING: NO. Eastern cannot be allowed to interconnect with PLDT. The defense of Art.
XII Sec. 18 of the Constitution is misplaced because the former has no franchise to
operate a telephone system.

MAIN POINT: Art. XII Sec. 18 of the Constitution has been used to justify compulsory
interconnection of a private telephone company with a government telephone system.
However, in this case, the records do not show any urgency for another company,
especially a non-franchised one, to operate a similar facility for exactly the same people
without having spent a single centavo to build up the domestic system. The proposed
international gateway will not add a single telephone unit to existing phones in the country.
We fail to see how a non-franchised telephone system will improve telephone services in
the Philippines through the proposed scheme.

CASE NO. 1313- CHIONG


ARTICLE XII: SECTION 19. MONOPOLIES AND COMBINATIONS
ENERGY REGULATORY BOARD V. CA, 357 SCRA 30 (2001)

FACTS:
This is a consolidated petition filed by (1) Energy Regulatory Board (ERB) and (2)
Pilipinas Shell Petroleum Coporation (Shell), engaged in the business of importing crude
oil, refining the same and selling various petroleum products through a network of service
stations throughout the country, against Court of Appeals and Petroleum Distributors and
Services Corporation (PDSC) which owns and operates a Caltex service station at the
corner of the MIA and Domestic Roads in Pasay City.

Shell filed an application to the Bureau of Energy Utilization (BEU) for authority to relocate
its Shell Service Station at Tambo, Parañaque, Metro Manila, to Imelda Marcos Avenue
of the same municipality. PDSC filed an opposition to the application on the grounds that:
1.] there are adequate service stations attending to the motorists’ requirements in the
trading area covered by the application; 2.] ruinous competition will result from the
establishment of the proposed new service station; and 3.] there is a decline not an
increase in the volume of sales in the area. The BEU rendered a decision denying Shell’s
application on a finding that there was “no necessity for an additional petroleum products
retail outlet in Imelda Marcos Avenue, Parañaque.” Dissatisfied, Shell appealed to the
Office of Energy Affairs (OEA).

Meanwhile, Executive Order No. 172 was issued creating the Energy Regulatory Board
(ERB) and transferring to it the regulatory and adjudicatory functions of the BEU. Thus,
OEA remanded the case to the ERB. The latter board granted the application of Shell
which was opposed by PDSC and appealed to CA. The CA reversed the decision of ERB.
Hence, this petition.

ISSUE:
Whether the CA gravely erred in making findings of facts contrary to those of the ERB
whose findings were based on substantial evidence?

RULING:
Yes. Time and again this Court has ruled that in reviewing administrative decisions, the
findings of fact made therein must be respected as long as they are supported by
substantial evidence, even if not overwhelming or preponderant; that it is not for the
reviewing court to weigh the conflicting evidence, determine the credibility of the
witnesses or otherwise substitute its own judgment for that of the administrative agency
on the sufficiency of evidence; that the administrative decision in matters within the
executive jurisdiction can only be set aside on proof of grave abuse of discretion, fraud
or error of law. [26] Petitioner ERB is in a better position to resolve petitioner Shell’s
application, being primarily the agency possessing the necessary expertise on the
matter. The power to determine whether the building of a gasoline retail outlet in a trading
area would benefit public interest and the oil industry lies with the ERB not the appellate
courts.

In the hierarchy of evidentiary values, proof beyond reasonable doubt is at the highest
level, followed by clear and convincing evidence, preponderance of evidence and
substantial evidence, in that order. A litany of cases has consistently held that substantial
evidence is all that is needed to support an administrative finding of fact. It means such
relevant evidence as a reasonable mind might accept to support a conclusion

MAIN POINT: Only competition which is fair can release the creative forces of the market.
The objective of anti-trust law is “to assure a competitive economy based upon the belief
that through competition producers will strive to satisfy consumer wants at the lowest
price with the sacrifice of the fewest resources. Competition among producers allows
consumers to bid for goods and services and, thus matches their desires with society’s
opportunity costs.” Additionally, the market system relies on the consumer to decide what
and how much shall be produced, and on competition, among producers who will
manufacture it.

CASE NO. 1314 - LAKBAO


ARTICLE XII: SECTION 19. MONOPOLIES AND COMBINATIONS
GARCIA V. EXECUTIVE SECRETARY

FACTS: For the second time, petitioner Enrique T. Garcia, Jr. (petitioner Garcia) asks
this Court to examine the constitutionality of Section 19 of Republic Act No. 8479 (R.A.
No. 8479), otherwise known as the Oil Deregulation Law of 1998) through this petition for
certiorari .1 He raises once again before us the propriety of implementing full deregulation
by removing the system of price controls in the local downstream oil industry - a matter
that we have ruled upon in the past. Nonetheless, petitioner Garcia again sought to
declare the new oil deregulation law unconstitutional on the ground that it violated Article
XII, Section 19 of the Constitution.

ISSUE: Whether Section 19 of R.A. No. 8479 as unconstitutional for contravening Section
19, Article XII of the Constitution.

RULING: In asking the Court to declare Section 19 of R.A. No. 8479 as unconstitutional
for contravening Section 19, Article XII of the Constitution, petitioner Garcia invokes the
exercise by this Court of its power of judicial review, which power is expressly recognized
under Section 4(2), Article VIII of the Constitution. The power of judicial review is the
power of the courts to test the validity of executive and legislative acts for their conformity
with the Constitution. Through such power, the judiciary enforces and upholds the
supremacy of the Constitution. Petitioner Garcia's issues fit snugly into the political
question mold, as he insists that by adopting a policy of full deregulation through the
removal of price controls at a time when an oligopoly still exists, Section 19 of R.A. No.
8479 contravenes the Constitutional directive to regulate or prohibit monopolies 15 under
Article XII, Section 19 of the Constitution. This Section states:
The State shall regulate or prohibit monopolies when the public interest so requires. No
combinations in restraint of trade or unfair competition shall be allowed.
Read correctly, this constitutional provision does not declare an outright prohibition of
monopolies. It simply allows the State to act "when public interest so requires"; even then,
no outright prohibition is mandated, as the State may choose to regulate rather than to
prohibit. Two elements must concur before a monopoly may be regulated or prohibited:
1. There in fact exists a monopoly or an oligopoly, and
2. Public interest requires its regulation or prohibition.
Whether a monopoly exists is a question of fact. On the other hand, the questions of (1)
what public interest requires and (2) what the State reaction shall be essentially require
the exercise of discretion on the part of the State. Stripped to its core, what petitioner
Garcia raises as an issue is the propriety of immediately and fully deregulating the oil
industry. Such determination essentially dwells on the soundness or wisdom of the timing
and manner of the deregulation Congress wants to implement through R.A. No. 8497.
Quite clearly, the issue is not for us to resolve; we cannot rule on when and to what extent
deregulation should take place without passing upon the wisdom of the policy of
deregulation that Congress has decided upon. To use the words of Baker v. Carr,16 the
ruling that petitioner Garcia asks requires "an initial policy determination of a kind clearly
for non-judicial discretion"; the branch of government that was given by the people the
full discretionary authority to formulate the policy is the legislative department.

MP: Article XII, Section 19 of the Constitution. This Section states:


The State shall regulate or prohibit monopolies when the public interest so requires. No
combinations in restraint of trade or unfair competition shall be allowed. Read correctly,
this constitutional provision does not declare an outright prohibition of monopolies. It
simply allows the State to act "when public interest so requires"; even then, no outright
prohibition is mandated, as the State may choose to regulate rather than to prohibit. Two
elements must concur before a monopoly may be regulated or prohibited: 1. There in fact
exists a monopoly or an oligopoly, and 2. Public interest requires its regulation or
prohibition.

CASE NO. 1315 – SABTALUH


ARTICLE XII: SEC. 19 – MONOPOLIES AND COMBINATIONS
TATAD VS. SECRETARY OF DEPARTMENT OF ENERGY

FACTS: In March 1996, R.A. No. 8180 (Downstream Oil Industry Deregulation Act of
1996) was enacted in pursuance to the deregulation of the power and energy thrust under
R.A. 7638. Under the R.A. No. 8180, any person or entity was allowed to import and
market crude oil and petroleum products, and to lease or own and operate refineries and
other downstream oil facilities. Petitioner Francisco Tatad questions the constitutionality
of Section 5 of R.A. No. 8180 since the imposition of tarrif violates the equal protection
clause and bars the entry of others in the oil industry business.
In a separate petition (G.R. 127867), petitioners Edcel Lagman, Joker Arroyo, Enrique
Garcia, Wigberto Tanada, Flag Human Rights Foundation, Inc., Freedom from Debt
Coalition and Sanlakas argued that R.A. No. 8180, specifically Section 15 is
unconstitutional because it: (1) gives undue delegation of legislative power to the
President and the Secretary of Energy by not providing a determinate or determinable
standard to guide the Executive Branch in determining when to implement the full
deregulation of the downstream oil industry; (2) Executive Order No. 392, an order
declaring the implementation of the full deregulation of the downstream oil industry, is
arbitrary and unreasonable because it was enacted due to the alleged depletion of the
Oil Price Stabilization Plan- a condition not found in R.A. No. 8180; and (3) Section 15 of
R.A. No. 8180 and E.O. No. 392 allow the formation of a de facto cartel among Petron,
Caltex and Shell in violation of constitutional prohibition against monopolies,
combinations in restraint of trade and unfair competition.

ISSUES: Whether or not R.A. No. 8180 violates the constitutional prohibition against
monopolies, combinations in restraint of trade and unfair competition.

RULINGS: Yes, R.A. No. 8180 is unconstitutional. It violated Section 19, Article XII of the
Constitution prohibiting monopolies, combinations in restraint of trade and unfair
competition. The deregulation act only benefits Petron, Shell and Caltex, the three major
league players in the oil industry. The three key provisions of R.A 8180 that is to promote
free and unfair competition where shown to achieve the opposite result. The court ruled
that its provisions on tariff differential, stocking of inventories, and predatory pricing inhibit
fair competition, encourage monopolistic power. and interfere with the free interaction of
market forces.

MAINPOINT: Tariff differential, stocking of inventories, and predatory pricing inhibit fair
competition, encourage monopolistic power. and interfere with the free interaction of
market forces

CASE NO: 1316 - VARGAS


ARTICLE XII: SECTION 19. MONOPOLIES AND COMBINATIONS
EASTERN ASSURANCE V. LTFRB

FACTS: In its desire to improve public service and its assistance to the victims of road
accidents involving PUVs (public utility vehicles), the Land Transportation Franchising
and Regulatory Board conducted a thorough investigation on the sufficiency of existing
insurance policies for PUVs. In the course of its investigation, the Board discovered that
insurance coverage of PUVs was only P50,000.00 for the entire vehicle regardless of the
number of passengers or persons killed or injured.
The Board, then, undertook nationwide consultations among the transport operators and
insurance companies and held meetings with the officials of the Insurance Commission.

ISSUE: The assailed LTFRB circulars with [their] implementing circulars violate the
constitutional proscription against monopoly, combination in restraint of trade and unfair
competition.
RULING: No. While embracing free enterprise as an economic creed, the Constitution
does not totally prohibit the operation of monopolies. However, it mandates the State to
regulate them when public interest so requires.
Intense competition has led insurance companies/agents offering insurance policies for
public utility vehicles to resort to ruinous tactics to sell their services. Notorious agents of
these companies have engaged in predatory pricing -- selling the compulsory insurance
coverage at an unbelievable discount of sixty to eighty percent (60 to 80%) off the market
rate. The huge coverage and liability under the no-fault clause of the passenger accident
insurance are grossly disproportionate to the small premiums actually being paid.

MAIN POINT: Since petitioner has failed to show any cogent reason to strike down the
assailed Circulars, their implementation cannot be restrained. They may indeed adversely
affect its business, but the protection of the general welfare is of paramount importance.
Petitioner’s individual business interests must be subordinated to the benefit of the
greater number.

CASE NO. 1317 - TIPOE


ART. XII SEC. 19: MONOPOLIES AND COMBINATIONS
AVON V. LUNA

FACTS: Respondent and petitioner entered into a contract of “Supervisor’s Agreement”.


Sec. 5 of the agreement includes a restriction that the Supervisor shall sell or offer to
sell, display or promote only and exclusively products sold by the Company.

ISSUE: W/N such restriction is violative of Art. XII Sec. 19 of the Constitution for being an
“unreasonable restraint of trade or occupation."

RULING: NO. Restrictions upon trade may be upheld when not contrary to public welfare
and not greater than is necessary to afford a fair and reasonable protection to the party
in whose favor it is imposed. Even contracts which prohibit an employee from engaging
in business in competition with the employer are not necessarily void for being in restraint
of trade. The contrary would be most unfair to Avon. The other companies cannot ride on
and exploit the training and experience of the Avon sales force to sell and promote their
own products.

MAIN POINT: Contracts requiring exclusivity are not per se void. Each contract must be
viewed vis-à-vis all the circumstances surrounding such agreement in deciding whether
a restrictive practice should be prohibited as imposing an unreasonable restraint on
competition.

You might also like